100% found this document useful (2 votes)
3K views125 pages

Rosh Review

This document discusses several medical conditions including: - Generalized anxiety disorder which is characterized by anxiety lasting over 6 months unrelated to specific events. Treatment includes CBT, SSRIs, SNRIs. - Adjustment disorder which involves emotional symptoms following an identifiable stressor lasting less than 6 months. Treatment includes CBT, SSRIs. - Persistent depressive disorder which is a milder form of depression lasting at least 2 years.

Uploaded by

Prince Du
Copyright
© © All Rights Reserved
We take content rights seriously. If you suspect this is your content, claim it here.
Available Formats
Download as DOCX, PDF, TXT or read online on Scribd
100% found this document useful (2 votes)
3K views125 pages

Rosh Review

This document discusses several medical conditions including: - Generalized anxiety disorder which is characterized by anxiety lasting over 6 months unrelated to specific events. Treatment includes CBT, SSRIs, SNRIs. - Adjustment disorder which involves emotional symptoms following an identifiable stressor lasting less than 6 months. Treatment includes CBT, SSRIs. - Persistent depressive disorder which is a milder form of depression lasting at least 2 years.

Uploaded by

Prince Du
Copyright
© © All Rights Reserved
We take content rights seriously. If you suspect this is your content, claim it here.
Available Formats
Download as DOCX, PDF, TXT or read online on Scribd
You are on page 1/ 125

They will give you a perfect candidate for aspirin forMI PREVENTION BUT BUT THEY will hve

history of ulcers and the answer will be something else like screen for diabetes

Generalized anxiety
disorder
Anxiety lasting > 6 months unrelated to a specific person, situation, or event. Associated with
restlessness, irritability, sleep disturbance, fatigue, muscle tension, difficulty concentrating.
Treatment: CBT, SSRIs, SNRIs are first line. Buspirone, TCAs, benzodiazepines are second line.
Adjustment disorder—emotional symptoms (anxiety, depression) causing impairment following
an identifiable psychosocial stressor (eg, divorce, illness) and lasting < 6 months (> 6 months in
presence of chronic stressor). Treatment: CBT, SSRIs.

`
Persistent depressive disorder (dysthymia)—
depression, often milder, lasting at least
2 years.
SIG E CAPS:
ƒ.Depressed mood
ƒ. Sleep disturbance
ƒ. Loss of Interest (anhedonia)
ƒ.Guilt or feelings of worthlessness
ƒ. Energy loss and fatigue
ƒ.Concentration problems
ƒ. Appetite/weight changes
ƒ. Psychomotor retardation or agitation
ƒ. Suicidal ideations
Patients with depression typically

calcium gluconate for cardioprotection

All pregnant women should be screened by urine culture for asymptomatic bacteriuria between 12
and 16 weeks' gestation and be offered GBS screening by vaginorectal culture at 35 to 37 weeks'
gestation.

At which gestational age should Rhogam be administered to an Rh neg woman?

Answer: 28 weeks.

What are the 3 values used to diagnose diabetes mellitus type 2?

Answer: Need 2: Fasting blood sugar > 126 mg/dL, HBA1c of 6.5%, glucose tolerance
test results > 200 mg/dL or a random plasma glucose level > 200mg/dL + symptoms of
hyperglycemia.

Question: What is the treatment for acute hypoparathyroidism?

Answer: Intravenous calcium gluconat

ACE inhibitors and angiotensin-receptor blockers (ARB) are contraindicated


during pregnancy due to their teratogenic effect!
Angiotensin-converting enzyme inhibitors such as lisinopril (B) and angiotensin receptor
blockers such as losartan (C) are contraindicated during pregnancy because they are
associated with significant fetal renal abnormalities.

Mineralocorticoid receptor antagonists such as spironolactone (D) cross the placenta and have
never been proven safe in pregnancy. The anti-androgenic activity of spironolactone is also a
concern, particularly in male fetuses.

Deep Vein Thrombosis (DVT)

ost commonly caused by stasis, hypercoagulable state, trauma (Virchow's triad)

Squamous Cell Carcinoma

 Diagnosis is made by clinical exam, skin biopsy to confirm

Chronic myelogenous leukemia (CML) (B) is characterized by an increased proliferation of the


granulocytic cell line and is characterized by the presence of the Philadelphia chromosome.
Splenomegaly is the most common physical finding in patients with chronic myelogenous leukemia.

Chronic lymphocytic leukemia (CLL) is a monoclonal disorder characterized by a progressive


accumulation of functionally incompetent lymphocytes

hat medications should be avoided in patients with glaucoma?

Answer: Decongestants and anticholinergic medications


The patient likely has Munchausen syndrome. This is a psychiatric factitious disorder in which
the individual feigns disease in an attempt to gain attention, sympathy, or reassurance. There
is often a longstanding history of doctor and hospital shopping, frequent hospitalizations, and
extensive medical records. They are generally well spoken, intelligent and able to communicate in
medical jargon. They often present on the weekend and after regular office hours in an attempt to
limit access to medical records and personal physicians. Individuals often want to be admitted to the
hospital and once admitted are difficult to discharge home. The normal serum lactate and lack of a
post-ictal state essentially rule out true seizure activity and support the fact the patient is faking his
seizure episodes.

The patient likely has Munchausen syndrome. This is a psychiatric factitious disorder in which
the individual feigns disease in an attempt to gain attention, sympathy, or reassurance. There
is often a longstanding history of doctor and hospital shopping, frequent hospitalizations, and
extensive medical records. They are generally well spoken, intelligent and able to communicate in
medical jargon. They often present on the weekend and after regular office hours in an attempt to
limit access to medical records and personal physicians. Individuals often want to be admitted to the
hospital and once admitted are difficult to discharge home. The normal serum lactate and lack of a
post-ictal state essentially rule out true seizure activity and support the fact the patient is faking his
seizure episodes.
Question: What is the first line intravenous antibiotic therapy for patients with
endometritis?

Answer: Clindamycin and gentamicin.

Endometritis

 Patient will be a woman 2 - 3 days post c-section

 Complaining of fever, abdominal pain, foul smelling lochia


Leukocytosis

The patient should have an ultrasound to rule out retained products of conception. She should be
started on intravenous broad-spectrum antibiotics and admitted to the hospital.

Perotonsillar absences=needle aspiration

Slipped capital femoral epiphysis (C) is the most common hip disorder in adolescents and occurs
mostly in obese patients who present with a limp and are unable to bear weight due to pain in the
hip, groin or knee. Legg-Calvé-Perthes disease (A) is avascular necrosis of the proximal femoral
head that occurs in children aged four to ten years of age. It presents with an intermittent limp
especially after exertion, with mild or intermittent pain in the anterior part of the thigh. Septic
arthritis (B) presents with acute onset of monoarticular excruciating joint pain often with spasm and
guarding, erythema, heat, and immobility.

The essential feature of depersonalization disorder (A) is the persistent or recurrent feeling of
detachment or estrangement from one's own self. The patient generally has intact memory for life
history, but experiences repeated states of depersonalization and derealization. With dissociative
identity disorder (C), the patient has two or more distinct personality states with at least two of
these identities recurrently taking control of the person's behavior. Post-traumatic stress disorder
(D) is characterized by amnesia for a traumatic event and symptoms of hyperarousal, avoidance and
dissociation. Depersonalization disorder, dissociative identity disorder and post-traumatic stress
disorder all share elements of dissociative fugue but are not characterized by wandering and loss of
autobiographical memory.

Administer subcutaneous sumatriptan and provide oxygen (B) is the first-line treatment for
an acute cluster headache attack. Ordering a brain MRI (C) is an appropriate step in
management in patients with a headache who present with focal neurologic symptoms, an acute
headache in the setting of trauma, are greater than 40 years old, have had a significant change
in the nature of the headaches, or have a worsening headache despite pharmacologic therapy.

post-traumatic stress disorder, which is diagnosed after four weeks.

Disc herniation (B) typically presents with pain on lumbar flexion (not extension). Additionally,
neurologic deficits can also occur with disc herniation compared to spondylolisthesis. A
vertebral fracture (D) typically results from trauma and is caused by falls, sudden deceleration
resulting from a motor vehicle collision, or direct injury. A vertebral fracture is not likely in this
case, due to the lack of a traumatic event and only mild tenderness elicited on palpation.

Radiating lower back pain and pain with lumbar hyperextension (> three weeks) without
history of a traumatic event suggests spondylolisthesis. Spondylolisthesis results from
anterior slippage of the spine forward, leaving posterior elements behind. Pain with lumbar
extension is the most common physical exam finding. Patients typically present with activity-
related pain. Diagnosis is typically made based on clinical exam findings and confirmed by
radiographs.

 Minimal change disease: children, preceded by URI, rx: steroids

 Focal segmental glomerulosclerosis: African-Americans, HIV/IVDA

 Membranous nephropathy: Caucasians, HBV, HCV, SLE, gold, penicillamine, malignancy


All women regardless of risk factors should begin screening by age 65. Men are not routinely
screened for osteoporosis unless risk factors are present.

A spermatocele (C) is a painless, sperm-filled mass separate from the testis. It is typically located
superior and posterior to the testis, is freely movable and transilluminates easily.

Epididymitis (A) is an acute infection of the epididymis and the patient usually presents acutely ill
with a fever and an exquisitely tender epididymis.

permatocele (C) is a painless, sperm-filled mass separate from the testis. It is typically located superior
and posterior to the testis, is freely movable and transilluminates easily.
GRowing pains (B) are defined as pain awakening a child at night (or from naps) who is
otherwise without any manifestation of musculoskeletal problems. If the child appears ill,
complains of pain during the day or with activity, or if the pain worsens or persists, the diagnosis
is unlikely to be growing pain

Legg-Calve-Perthes disease (C) is a syndrome of idiopathic avascular necrosis of the hip that
occurs in children between the ages of 3 and 12 years. Children with Legg-Calve-Perthes
disease present with a limp of insidious onset and a Trendelenburg gait. Pain, if present, is mild
and often referred to the anteromedial thigh or knee. On examination, patients may have limited
internal rotation and abduction of the hip. The classic presentation of slipped capital femoral
epiphysis (D) is that of an obese adolescent with a complaint of nonradiating, dull, aching pain
in the hip, groin, thigh, or knee, and no history of preceding trauma.

As a basic rule, in a patient with a clinically suspected scaphoid fracture but negative initial
radiographs it is reasonable to apply a short arm thumb spica splint and reevaluate the patient in two
weeks. If a cast is not applied initially, the fracture can worsen over the following months

Which malignancy is a sequelae of hepatitis B infection?

Answer: Hepatocellular carcinoma.


What is the definition of the number needed to treat?

Answer: The number of patients in a population that need to be treated in order to make one good
outcome.

References:

The sensitivity (C) is defined as the fraction of patients who truly have a disease and are identified
as such by a positive test result. A sensitive test is good at ruling out disease as patients with a
negative test result are not likely to have the disease. The specificity (D) of a test refers to the
fraction of patients without a specific disease who have a negative test result. Therefore a positive
result of a test with a high specificity is good at ruling in the disease.

Acute lymphoblastic leukemia (ALL) (A) is a leukemia most often seen in children with
lymphoblasts predominating. Chronic lymphocytic leukemia (CLL) (C) is a leukemia that presents
in older adults

Chronic myelogenous leukemia (CML) (D) presents in adults with a cell lineage of mature cell
lines with a marked increase in basophils and eosinophils.

Warfarin cannot be used alone in prevention of DVT prophylaxis (D). It must be bridged with
either heparin or LMWH such as enoxaparin for a minimum of five days in order to prevent warfarin
skin necrosis and to reach a target INR of 2-3.

Considered safe in pregnancy if Contraindicated during pregnancy or Special


otherwise indicated safety not established recommendation
s pertain

Tetanus and diphtheria toxoids BCG*Measles*Mumps*Rubella*Varicella*


(Td)Hepatitis
BInfluenzaMeningococcalRabie
s

What are the characteristic physical exam findings of a varicocele?

Answer: A scrotal mass described as a "bag of worms" that decompresses and


disappears in the recumbent position.

secondary syphilis (C) may closely resemble pityriasis rosea. However, secondary syphilis
often presents with red-brown macules on the palms and soles

Meconium ileus is characterized by obstruction of the bowel by meconium, or infant stool, in a


newborn infant. Infants generally present during the first three days of life with abdominal distension
with or without vomiting and failure to pass meconium. It is the presenting problem in 10 to 20% of
newborns with cystic fibrosis.
. Infants with phenylketonuria (C) are asymptomatic prior to the initiation of feeds containing
phenylalanine such as breast milk or standard infant formula and can eventually lead to mental
retardation.

Patients with a glomerular filtration rate between 30 and 60 mL/min or heart failure should
discontinue metformin

The patient’s laboratory results demonstrate a macrocytic anemia. Of the medications she is
taking, methotrexate is the most likely to cause a macrocytic anemia. Methotrexate is a folate
antimetabolite that irreversibly binds to and inhibits dihydrofolate reductase.

What are the side effects of estrogen therapy in osteoporosis?

Answer: Increased risk for breast and endometrial cancer.


Myxedema coma

Puffiness of the hands and face, a thickened nose, swollen lips and an enlarged tongue may occur
secondary to nonpitting edema with abnormal deposits of mucin in the skin and other tissues,
referred to as myxedema.

Myxedema Coma

 Hypothyroidism exacerbation → ↓ metabolic state + AMS

 Stupor

 Hypoventilation

 Hypotension, bradycardia

 Rx: IV levothyroxine

 High mortality
The predominant manifestation of adrenal crisis (A) is shock, but the patients often have
nonspecific symptoms such as anorexia, nausea, vomiting, abdominal pain, weakness, fatigue,
lethargy and fever. Often hyperpigmentation is present, not puffiness.

Electrolyte imbalances should be identified and addressed immediately. Hyperkalemia is often


present in patients with acute kidney injury and an electrocardiogram should be done to detect
any abnormalities in cardiac function, such as tall, peaked T waves. In patients with severe
hyperkalemia, five to 10 units of regular insulin along with dextrose should be administered
intravenously to help shift potassium into the cells. Calcium gluconate should also be given to
patients with electrocardiogram changes in order to stabilize the cardiac membrane and to
reduce the risk of dysrhythmias.
A major depressive episode is a period lasting at least two weeks, with five or more of the
following symptoms: depressed mood, anhedonia, insomnia or hypersomnia, change in appetite
or weight, psychomotor retardation or agitation, low energy, poor concentration, thoughts of
worthlessness or guilt, and recurrent thoughts about death or suicide; at least one of the
symptoms must be depressed mood or anhedonia. Treatment of major depression with a
combination of pharmacotherapy and psychotherapy is more efficacious than either alone

Dextromethorphan used to treat a cold can cause serotonin syndrome

Cholinergic—nausea, vomiting, diarrhea, blurry yellow vision (think


DIGOXIN ADV ERSE EFFECTS
van Gogh), arrhythmias, AV
block.
Can lead to hyperkalemia, which indicates poor prognosis.

NORMALLY conduction at AV node and depression of SA node).

Bronchiolitis is characterized by upper respiratory symptoms followed by a lower respiratory


infection occurring in children < 2 years of age. Bronchiolitis results from a viral infection of the
terminal bronchiolar epithelial cells. The infection causes inflammation and edema in the small
bronchi and bronchioles which leads to atelectasis. Patients with bronchiolitis presents with fever,
cough, and respiratory distress. A history and physical exam is sufficient to diagnose nonsevere
bronchiolitis which may include an increased respiratory rate, retractions, wheezing, or crackles.

Acute Bronchitis

 Patient will be complaining of a productive cough for less than a week

 Most commonly caused by viruses

 Treatment is symptomatic and bronchodilators

 Comments:

 Most common cause of minor hemoptysis

 Routine ABX therapy not indicated

 Question: What virus is the most common cause of bronchiolitis?

 Answer: Respiratory syncytial virus (RSV).

 hat are the two diagnostic modalities used for ovarian cancer?

 Answer: Transvaginal ultrasonography and CA-125 testing.

s. This recommendation does not apply to women who have an increased-risk family history of
ovarian cancer. Increased-risk family history is defined as 1) having two or more first- or second-
degree relatives with a history of ovarian cancer or a combination of breast and ovarian cancer,
or 2) a woman of Ashkenazi Jewish descent with a first-degree relative (or two second-
degree relatives on the same side of the family) with breast or ovarian cancer. Such
women with an increased-risk family history should be referred for genetic counseling in addition
to screening for ovarian cancer.

Nongenetic risk factors for ovarian cancer include increased age, obesity or weight gain (A),
and hormone replacement therapy for more than five years after menopause (C). Other
risk factors such as infertility treatments and smoking are equivocal. None of these risk factors
warrant screening for ovarian cancer in an asymptomatic woman

Most patients with aspiration pneumonia acquired in the community have a mixed bacterial
infection that includes strict anaerobes and facultative anaerobes such as oral streptococci.
Intravenous ampicillin-sulbactam is first-line therap

Question: Normal- pressure hydrocephalus is characterized by which triad of


symptoms?

Answer: Personality changes (Wacky), urinary incontinence (Wet) and an unbalanced


gait (Wobbly).

Lewy body dementia is progressive degenerative dementia of unknown etiology. It is a clinical


diagnosis that includes memory deficits, reduced alertness as well as visual hallucinations
and parkinsonian motor features, which include bradykinesia, resting tremor, rigidity, and gait
difficulty.

Acetylcholinesterase inhibitors are first line treatment in Lewy body dementia as acetylcholine is
decreased in the brain in this disorder. When cholinesterase inhibitors are ineffective, most experts
recommend atypical neuroleptics such as clozapine, quetiapine, or aripiprazole.

Bilirubin is a product of heme catabolism. Increased total serum or plasma bilirubin puts
neonates at risk of bilirubin-induced neurologic dysfunction caused by bilirubin crossing the
blood-brain barrier and binding to brain tissue. Kernicterus is the chronic and permanent
sequela of bilirubin-induced neurologic dysfunction. Suboptimal breastfeeding is associated
with an increased risk of jaundice and kernicterus. Breastfeeding failure jaundice occurs
within the first week of life due to inadequate intake and fluid loss resulting in hypovolemia,
causing hyperbilirubinemia and hypernatremia.

Bilirubin is a product of heme catabolism. Increased total serum or plasma bilirubin puts
neonates at risk of bilirubin-induced neurologic dysfunction caused by bilirubin crossing the
blood-brain barrier and binding to brain tissue. Kernicterus is the chronic and permanent
sequela of bilirubin-induced neurologic dysfunction. Suboptimal breastfeeding is associated
with an increased risk of jaundice and kernicterus. Breastfeeding failure jaundice occurs
within the first week of life due to inadequate intake and fluid loss resulting in hypovolemia,
causing hyperbilirubinemia and hypernatremia.

Crigler-Najjar
syndrome, type I
Absent UDP-glucuronosyltransferase. Presents
early in life; patients die within a few years.
Findings: jaundice, kernicterus (bilirubin
deposition in brain),  unconjugated bilirubin.
Treatment: plasmapheresis and phototherapy.
Type II is less severe and responds to

phenobarbital, which  liver enzyme synthesis.

hat unique organism is associated with osteomyelitis in a sickle cell patient?

Answer: Salmonella
Obtain at least 2 sets of blood cultures to rule out a bacteremic origin of the septic joint.

The patient is suffering from acute cholangitis, which occurs when there is concomitant
common bile duct obstruction (often secondary to a stone) and retrograde ascending bacterial
infection.

Acute Cholangitis

 Patient will be complaining of right upper quadrant pain, jaundice, fever (Charcot’s
triad)

 Diagnosis is made by RUQ ultrasound, CT scan, or ERCP (Gold standard)

 Most commonly caused by choledocholithiasis leading to bacterial infection, E.coli

 Treatment is antibiotics, definitive treatment is ERCP with antibiotics typically an adjunct

 Comments: Charcot's triad + hypotension and AMS = Reynolds pentad, acute


obstruction
Patients with acute cholecystitis (B) may present very similarly to patients with acute cholangitis,
however patients with cholecystitis will not have jaundice and elevated bilirubin levels.

hat medication should be avoided in the active phase of labor in women vaginally
delivering a fetus in breech presentation?

Answer: Oxytocin.

Carboprost (A) is a prostaglandin analog that can cause bronchoconstriction and should not be
used for termination of pregnancy, cervical ripening, induction of labor, or control of uterine
hemorrhage in women with asthma. Carboprost is not indicated in the treatment of head
entrapment

Question: What is the CHADS2 score used for?

Answer: Predicting the risk of stroke in patients with atrial fibrillation.


In women, androgenic steroids should be suspected in patients who present with irregular
menstrual cycles, hirsutism, acne, breast atrophy, temporal hair recession, deepening of the
voice, clitoromegaly, increased muscle mass, and a decrease in body fat.

Performance-enhancing drugs are often used by athletes to enhance their performance and
to appear leaner and more muscular

A high serum calcitonin level (A) is a characteristic feature of medullary thyroid cance

Thyroid-stimulating hormone (TSH) screening for women in the first trimester is


recommended for recurrent miscarriage or preterm delivery, multiple prior pregnancies

Candida albicans Oral thrush Scrapable white plaque, pseudohyphae on


microscopy
EBV Oral hairy leukoplakia Unscrapable white plaque on lateral tongue

Oral Lichen Planus (D) lesions are violaceous, flat-topped, angulated papules with very fine white
streaks (Wickham striae) on the surface of the lips, tongues, and buccal mucosa. The mucosal
lesions of Lichen Planus have a lacy white network overlying them that may be confused with
leukoplakia. Oral Lichen Planus require biopsy and often direct immunofluorescence for diagnosis.

All adults should receive thw td booster every 10 years

If no histpry of tdap od history of vaccination. Given tdap then give td booster every 10 years

Sequential Vccination with the 13 valent pneumococcal conjugate cavvine PCV13 followed at a later time
by the 23 valent pneumococcal Ppolysaccharide vaccine PPSV23 is recommended for all adults age
greater than 65 and for those less than 65 with vertain very high risk comorbid conditions

For adults less than 65 with other chronic medical conditions that increase the risk of invasive
pneumococcal disease e.g heart or lung disease, diabetes , smoking or chronic liver disease. PPSV23
alone is recommended followed by sequential PCV13 and ppsv23 AT AGE 65

Patients with chronic liver disease who contract viral hepatitis are at increased risk for complications. For
this reason they should receive vaccination against hepatitis Aand B unless they have documented
evidence of immunity

Malignant hyperthermia (A) occurs with the use of certain anesthetic agents (halothane and
succinylcholine) and manifests as severe muscle rigidity and hyperthermia.

Treatment of pelvic inflammatory disease typically requires antimicrobial coverage against the
most common pathogens, including Neisseria gonorrhoeae and Chlamydia trachomatis in which
case ceftriaxone plus doxycycline (D) would be the appropriate treatment.
The Haemophilus influenzae type B vaccine, often called Hib vaccine, is a vaccine used to
prevent Haemophilus influenzae type b (Hib) infection.[1] In countries that include it as a routine
vaccine, rates of severe Hib infections have decreased more than 90%.[1] It has therefore resulted in
a decrease in the rate of meningitis, pneumonia, and epiglottitis.[1]
It is recommended by both the World Health Organization and Centers for Disease Control and
Prevention.[1][2] Two or three doses should be given before six months of age. [1] In the United States a
fourth dose is recommended between 12 and 15 months of age. [3] The first dose is recommended
around six weeks of age with at least four weeks between dose

Question: Pick’s disease is characterized by which type of dementia?

Answer: Frontotemporal.

Frontotemporal dementia (C) is a type of cortical dementia characterized by behavioral and


personality disorders more than cognitive issues.

: Triptans, DHE: contraindicated in HTN or CV disease

NSAIDS protective for gastric adenocarcinoma

Traveler's Diarrhea

 Patient with a history of recent travel

 Complaining of abrupt onset of watery diarrhea, nausea, and abdominal cramping

 Most commonly caused by enterotoxigenic Escherichia coli (ETEC)

 Treatment is rehydration and ciprofloxacin or azithromycin (pregnant women and


children)

Metronidazole (C) is the treatment of choice when parasitic infection like Giardia or Entamoeba
histolytica is suspected. However bacteria are the most common cause of traveler’s diarrhea and
ciprofloxacin is recommended as the first-line agen

Bacterial enterotoxins block passive sodium and water resorption and stimulate sodium excretion,
resulting in a net loss of fluid.

Laxatives are an example of increased osmotic load to the colon causing diarrhea.

Most cases of traveler’s diarrhea are self-limited. The treatment for traveler’s diarrhea includes oral
rehydration with glucose-containing fluids, ciprofloxacin, and consideration of anti-motility agents like
loperamide. Anti-motility agents should not be used in patients with bloody diarrhea or suspected
inflammatory diarrhea owing to risk of complications like prolonged fever, toxic megacolon, or
hemolytic uremic syndrome in children.

Primary biliary cirrhosis is characterized by a T-lymphocyte-mediated attack on small intralobular


bile ducts of the liver. A continuous assault on the bile duct epithelial cells leads to their gradual
destruction and eventual disappearance and eventually results in cirrhosis and liver failure. The
diagnosis should be considered in the patient, particularly a woman, who complains of unexplained
itching, fatigue, jaundice, or unexplained weight loss with right-upper-quadrant discomfort, and
whose serum alkaline phosphatase is unaccountably elevated.ANTI MITOCHONDRIAL
ANTIBODIES

What are some diseases associated with primary biliary cirrhosis?

Answer: Sjögren's syndrome, arthritis, and Raynaud phenomenon. Sicca symptoms are
frequent.

Vs primary sclerosing cholngitis which has p-anca

According to the American Academy of Otolaryngology–Head and Neck Foundation, a CT scan


(A) or magnetic imaging should not be done before examining the larynx by a laryngoscopy

Higher the anion gap=metabolic acidosis

Metabolic acidosis
• Blood pH 7.3 and serum HCO3

− 15 mEq/L

Ptosis, dilated pupil, and displacement of the eye downward laterally (D) are features of a cranial
nerve III palsy

: What congenital heart anomaly is a possible result of lithium treatment and a reason it
is contraindicated in pregnant patients?

Answer: Ebstein's anomaly, a congenital malformation in which the septal and posterior
leaflets of the tricuspid valve are displaced towards the apex of the right ventricle of the
heart.

LMNOP—Lithium side effects:


Movement (tremor)
Nephrogenic diabetes insipidus
HypOthyroidism

Pregnancy problems

Causes Ebstein anomaly in newborn if taken


by pregnant mother. Narrow therapeutic
window requires close monitoring of serum

levels.

Risperidone (D) is second-generation atypical antipsychotic mainly used in Schizophrenia,


Schizoaffective Disorder, and the mixed and manic states of Bipolar Disorder. Risperidone toxicity
may result in Neuroleptic Malignant Syndrome (NMS). NMS is a rare, but life-threatening,
idiosyncratic reaction to neuroleptic medications that is characterized by fever, muscular rigidity,
altered mental status, and autonomic dysfunction
s of Bipolar Disorder. Maintenance therapy reduces the frequency of manic episodes and diminishes
the intensity of those episodes which may occur. Typical symptoms of mania include pressure of
speech, motor hyperactivity, reduced need for sleep, flight of ideas, grandiosity, elation, poor
judgment, aggressiveness, and possibly hostility

Pneumonia is the most common serious complication of influenza infection in the adult population,
particularly in the elderly and in those with chronic medical conditions. This includes both a primary
viral pneumonia and secondary bacterial pneumonia

A 33-year-old woman with no past medical history presents with a cough and fever for one week.
Her cough is productive of green sputum. Auscultation of the chest is normal. Vital signs are: T
100.2°F, BP 120/72, HR 88, RR 12, and 100% saturation on room air. She is otherwise well-
appearing. What is the most appropriate plan?

Bronchitis

 Patient will be complaining of a mucopurulent cough for more than 5 days

 PE will show wheezing and rhonchi

 CXR will show thickening of the bronchial walls in the lower lobes

 Most commonly caused by viruses

 Treatment is symptomatic relief

Treatment of bronchitis is initially supportive. The goals of management are to alleviate the
patient’s symptoms. Options for treatment include antitussive agents, non-steroidal anti-
inflammatories, mucolytics, and possibly short-acting beta-agonists if the patient is experiencing
bronchial inflammation causing wheezing or excessive coughing.

Noninvasive diagnosis of arterial disease


An ABI of 0.91 to 1.30 is normal, and no further testing is necessary (B) in asymptomatic
patients. An ABI ≤ 0.90 is diagnostic of arterial obstruction (C). An ABI > 1.30 suggests the
presence of calcified vessels

However, in patients with a normal ABI (0.91 to 1.30) who have symptoms of claudication, exercise
testing is recommended to further evaluate for arterial obstruction or to suggest an alternative
etiology for the symptoms.

 Minimal change disease: children, preceded by URI, rx: steroids

 Focal segmental glomerulosclerosis: African-Americans, HIV/IVDA

 Membranous nephropathy: Caucasians, HBV, HCV, SLE, gold, penicillamine, malignancy


Proteinuria, hypoalbuminemia, edema and hyperlipidemia are characteristic clinical features
of nephrotic syndrome. Nephrotic syndrome is caused by renal diseases that increase the
permeability across the glomerular filtration barrier.

Question: How does aspirin inactivate platelets?

Answer: Aspirin inhibits the synthesis of thromboxane A2 (TxA2), a potent stimulator of


platelet aggregation.

Slipped Capital Femoral Epiphysis (SCFE)

 Patient will be an obese man 12 - 16-years-old

 Complaining of a progressive limp and knee pain

 PE will show loss of hip internal rotation

 Xray will show "scoop of ice cream slipping off an ice cream cone"

 Diagnosis is made by AP and frog-lateral X-rays

 Treatment is non-weight bearing and urgent orthopedic consultation

Patients with SCFE present with progressive limp and pain localized to the groin, thigh or knee.

Legg-Calvé-Perthes disease (A) or idiopathic avascular necrosis of the proximal femoral epiphysis,
most commonly occurs in male children between the ages of 3 and 12 years with a peak between 5
and 7 years of age. It often presents with limp that has an insidious or stuttering onset. The
associated pain is generally mild, refers to the groin, anteriomedial aspect of the thigh or knee, is
often relieved by rest, and generally worse at the end of the day.

Bronchiolitis is a clinical syndrome that most commonly occurs in children less than 2 years of
age. It is characterized by upper respiratory symptoms leading to lower respiratory infection with
inflammation, which results in wheezing or crackles. It typically occurs with primary infection or
reinfection with a viral pathogen. Infants and children with moderate to severe respiratory
distress (e.g., nasal flaring, retractions, grunting, respiratory rate > 70 breaths per minute,
dyspnea, cyanosis) usually require hospitalization for supportive care and monitoring.
Additional indications for hospitalization include toxic appearance, poor feeding, lethargy,
apnea, and hypoxemia. Supportive care (maintenance of adequate hydration, relief of nasal
congestion, monitoring disease progression) and anticipatory guidance are the mainstays of
management

Transient synovitis (D) is a self-limited non-pyogenic inflammation of the synovium that most
commonly occurs in boys between the ages of three and six years of age. The exact etiology is
unknown although in many cases follows a mild upper respiratory infection. Children will
complain of hip pain, walk with a limp or refuse to bear weight. They are generally otherwise well.
Question: What medication must be initiated when prescribing oral isotretinoin?

Answer: Patients must be on a form of birth control before taking oral isotretinoin as it is
known to be a teratogen.

Topical antibiotics (C) are used predominantly for the treatment of mild to moderate inflammatory or
mixed acne. Clindamycin and erythromycin are the most studied. Oral antibiotics (A) are effective for the
treatment of moderate to severe acne. The best-studied antibiotics include tetracycline and
erythromycin. Oral isotretinoin (B) is reserved for treatment of severe, recalcitrant acne.

Comedones are noninflammatory acne lesions.

 Mild to moderate: topical retinoids, topical antibiotics, or benzoyl peroxide

 Moderate to severe: add oral antibiotics

 Severe: oral isotretinoin (Pregnancy class X, must have two forms of birth control)
Uncontrolled maternal hyperglycemia leads to increased fetal growth resulting in macrosomia
and fetal hyperinsulinemia, increasing the risk of neonatal hypoglycemia.

Question: The recombinant varicella zoster vaccine is recommended beginning at what


age?

Answer: Healthy adults 50 years of age and over.


Testosterone deficiency or hypogonadism can either be primary or secondary. Primary
hypogonadism results in dysfunction of the testes while secondary hypogonadism is the result of
disease of the hypothalamus or pituitary. In order to distinguish between the two, serum
concentrations of luteinizing hormone (LH) and follicle-stimulating hormone (FSH) must be
measured.
Since this patient was able to confirm presence of spontaneous erections, nocturnal penile
tumescence testing (NPT) (B) is not required. If a patient or his partner is not able to confirm the
presence of spontaneous erections, NPT can provide proof. Absence of nocturnal erections is
consistent with a vascular or neurologic cause.

A palpable cord (D)/tender cord is more common in superficial thrombophlebitis than venous
insufficiency.

Question: Contact dermatitis is in which hypersensitivity class?

Answer: Type IV hypersensitivity.

Question: What are the recommendations for child safety seats for children under 2
years of age?

Answer: Rear-facing child seats.


Bronchiolitis is characterized by upper respiratory symptoms followed by a lower respiratory
infection occurring in children < 2 years of age. Bronchiolitis results from a viral infection of the
terminal bronchiolar epithelial cells. The infection causes inflammation and edema in the small
bronchi and bronchioles which leads to atelectasis. Patients with bronchiolitis presents with fever,
cough, and respiratory distress.

Question: What virus is the most common cause of bronchiolitis?

Answer: Respiratory syncytial virus (RSV).

Aphasia and personality changes (A) is seen in patients that have had a stroke or in patients that
suffer from vascular dementia. Social disinhibition (D) is seen in frontotemporal dementia. Pick’s
disease is a form of frontotemporal dementia

Question: Normal- pressure hydrocephalus is characterized by which triad of


symptoms?

Answer: Personality changes (Wacky), urinary incontinence (Wet) and an unbalanced


gait (Wobbly).

Lewy body dementia=parkinson shit e.g bradykinesia etc

Pelvic inflammatory disease (PID) (D) the patient will have abdominal tenderness, adnexal
tenderness, cervical motion tenderness and an elevated temperature.

A spontaneous abortion, or miscarriage, is a pregnancy loss before the 20th week of


gestation. Risk factors for spontaneous abortion are advanced maternal age, previous
spontaneous abortion, and maternal smoking

Early cervical cancer is frequently asymptomatic, underscoring the importance of screening.


The most common symptoms at presentation are irregular or heavy vaginal bleeding or
postcoital bleeding. Some women present with a vaginal discharge that may be watery,
mucoid, or purulent and malodorous.

Any lesion that is raised, friable, or has the appearance of condyloma should be biopsied,
regardless of previous benign cervical cytology results

Cervicitis is an inflammation and irritation of the cervix. Symptoms of cervicitis can be


similar to vaginitis, with vaginal discharge, itching or pain with
intercourse. Cervicitiscan be caused by a sexually transmitted infection. Most common
are chlamydia and gonorrhea.

Vaginal warts can be treated by applying trichloroacetic acid to the lesion (A)

Since this patient is Rh(D) negative and the father is known to be Rh(D) positive, prophylaxis
with anti-D immune globulin is recommended in the third trimester and again after delivery.

Rh(D) typing and an antibody screen are recommended at the first prenatal visit. In Rh(D)-
negative women, the antibody screen may be repeated at 28 weeks of gestation and at the
time of delivery.
Endometrial Cancer

 Patient will be a post-menopausal woman

 Complaining of abnormal vaginal bleeding

 Diagnosis is made by transvaginal ultrasound or endometrial biopsy USMLE

 Most common type is adenocarcinoma

 Treatment is total abdominal hysterectomy and bilateral salpingo-oophorectomy (TAH-


BSO)

A repeat Papanicolaou smear (C) is used to diagnose cervical cancer but not endometrial
cancer.

Transvaginal ultrasound is the preferred initial diagnostic test of choice to evaluate painless
vaginal bleeding in a postmenopausal patient in order to rule out endometrial (uterine)
carcinoma. Transvaginal ultrasonography is used to measure the endometrial thickness, which
should be less than 5 mm in a healthy patient.

Question: Which hereditary cancer syndromes predispose a woman to an increased


risk of endometrial carcinoma?

Answer: Lynch and Cowden syndromes.

Acute Otitis Media

 Patient will be an infant or young child

 Complaining of ear pain, fever, URI symptoms

 PE will show TM erythema and decreased mobility of TM

 Most commonly caused by viral > bacterial (S. pneumoniae most common)

 Treatment is amoxicillin

Macrolides such as azithromycin (C)

Amoxicillin-clavulanate (B) is the first-line therapy for children with acute otitis media who are
treated with antibiotics and are at increased risk of beta-lactam resistance due to having received
a beta-lactam antibiotic in the previous 30 days, have concomitant purulent conjunctivitis, or have
a history of recurrent acute otitis media
Question: Which medication inhibits alcohol dehydrogenase (ADH)?

Answer: Fomepizole.

Acute toxicity from organophosphates (B) present with manifestations of cholinergic excess.
The dominant clinical features of acute cholinergic toxicity include bradycardia, miosis,
lacrimation, salivation, bronchorrhea, bronchospasm, urination, emesis, and diarrhea.

Question: What lab value needs to be closely monitored in a patient started on


warfarin?

Answer: International normalized ratio (INR).

Oral griseofulvin (A) has been used for the treatment of onychomycosis, but is not as effective
as terbinafine or itraconazole and requires at least four to six months of treatment

scaphoid fracture (C) is a fall on the outstretched hand with an extended, radially deviated wrist.
On physical exam, there may be mild wrist swelling or bruising and, possibly, fullness in the
anatomic snuffbox, suggesting a wrist effusion.

Distal radius fractures that are minimally displaced or impacted can be treated with immobilization
for four to eight weeks. Younger patients with good bone health and non-displaced fractures often
can be treated with a volar splint, whereas patients with minimal displacement or osteoporotic bone
should have the extra protection of a short arm cast. It is imperative to perform weekly radiography
for the first three weeks because even fractures that appear stable may displace during early
treatment, particularly in older patients. Traditionally, surgical treatment has been reserved for
displaced, irreducible fractures or reducible but unstable distal radial fractures.

6PD Deficiency

 Patient with a history of taking antimalarials, sulfonamides, nitrofurantoin, fava beans

Cluster headaches are treated based on empirical data since there is no clear understanding of the
mechanism that underlies the disease. The therapy is either for acute management aimed at
aborting an attack or prophylactic therapy during the cluster period. Initial treatment of an acute
cluster headache is with 100% oxygen with a nonrebreather mask. If the oxygen therapy does not
provide relief, subcutaneous sumatriptan can be used as an abortive therapy.

Question: What preventative therapy is recommended for patients with chronic cluster
headache?

Answer: Verapamil.

 The United States Preventive Services Task Force recommends routine screening for
sexually active women < 24 years of age, and in women > 24 years of age who are at
increased risk
Metronidazole (C) is used for the treatment of vaginal trichomoniasis and bacterial vaginosis, but is
ineffective against Chlamydia.
Chlamydia trachomatis is susceptible to macrolides and tetracyclines antibiotics, which is the most
common bacterial cause of sexually transmitted infections (STIs) in the United
States. Azithromycin may be given in a one-time oral dose of 1 gram. Doxycycline may also be
prescribed as 100 mg twice per day for seven days and should be avoided in pregnant individuals.

Febrile Seizure - Simple

 Patient will be a child 6 months to 5 years of age

 With a history of fever > 38º C

 Complaining of a single tonic-clonic seizure lasting < 15 mins

 The degree of fever, not the rate of temperature rise, is thought to be the precipitating
stimulus

 Treatment is supportive care. If lasting > 5 min, a dose of diazepam gel or suppository
into the rectum can be used
Somatization disorder (D) begins before 30 years of age and manifests as multiple,
unexplained physical complaints (four pain symptoms, two gastrointestinal symptoms, one sexual
symptom, and one neurologic symptom).

Aplastic crisis is characterized by an acute drop in hemoglobin level caused by a transient


arrest of erythropoiesis, leading to abrupt reductions in red cell precursors in the bone
marrow and a markedly reduced number of reticulocytes in the peripheral blood
(reticulocytes < 1.0%). Infection is the typical cause. Most cases in children follow infection with
human parvovirus B19, which specifically invades proliferating erythroid progenitors. Aplastic
crisis can result in a rapid and life-threatening drop in hemoglobin levels in patients with sickle
cell anemia caused by chronic hemolysis without the ability of the bone marrow to
compensate. Management of patients with sickle cell anemia often requires transfusion.

Acute chest syndrome (A) refers to a syndrome of fever, chest pain, hypoxemia, wheezing,
cough, or respiratory distress in the setting of a new pulmonary infiltrate and occurs in as many
as half of patients with sickle cell disease.

Legg-Calve-Perthes Disease

 Patient will be a boy 4-10-years-old

 Complaining of a unilateral, intermittent limp

 Most commonly caused by osteonecrosis of the proximal femoral head

Initiating a prandial bolus of insulin (C) would be the next step if the basal insulin was not
sufficient in accomplishing glycemic control. Prescribing a third oral agent (D) is unlikely to
provide the glycemic control this patients needs.
Treatment of patients with type 2 diabetes begins with diet, weight reduction, exercise, and
metformin therapy. These first-line interventions can result in normoglycemia when patients
are compliant and successful. If a patient has persistent hyperglycemia on metformin and a
glycated hemoglobin (A1C) > 8.5 percent, it is recommended to add a second oral or injectable
agent, including insulin, or switch to insulin. Initial therapy with insulin is indicated in patients
who present with A1C > 9.5%, fasting plasma glucose > 250 mg/dL, random glucose
consistently > 300 mg/dL, or ketonuria. For patients who are initiating insulin, starting with
basal, rather than prandial, insulin is recommended. This is predominantly due to greater
patient satisfaction and a lower risk of hypoglycemia. Basal insulin alone is often adequate
for glycemic control since postprandial glycemia can be managed by endogenous insulin
secretion.

Question: Patients who have a history of transfusion prior to what year require a one-
time HCV screening?

Answer: 1992.
All patients born between the years 1945 and 1965 are a part of the high-prevalence birth cohort and
should be screened. 53 and older

Although abnormal liver function tests may be an indication for HCV testing, normal liver function
tests should not prevent screening from being performed

When a newborn or young infant presents with fever and lethargy or irritability, meningitis is a
primary concern.

The widespread use of the H. influenzae type b (Hib) vaccine has caused a dramatic decrease in
the incidence of pediatric bacterial meningitis.

Asthma
 Asthma: airway inflammation + bronchial hyperresponsiveness + reversible airflow
obstruction
 PEF <50%: severe exacerbation
 Treatments:
 O2: maintain SpO2 > 88%
 ß-agonists: ↑ cAMP → bronchodilation
 Anticholinergics: ↓ bronchoconstriction
 Corticosteroids: ↓ inflammation, administer early
 Mg: severe exacerbations
 Non-invasive ventilation: ↓ work of breathing

 Mechanical ventilation:

 Objective: maximize expiratory time

 Low respiratory rate

 High inspiratory flow rate


 Maintain plateau pressure <30 cm H2O

 Permissive hypercapnia to avoid breath stacking

Question: What are the local and systemic side effects of inhaled corticosteroids?

Answer: Local: Cough, dysphonia, oropharyngeal candidiasis. Systemic: Adrenal suppression,


osteoporosis, skin thinning, easy bruising and cataracts.

Long acting beta agonist inhalers (C) should not be used in place of anti-inflammatory therapy.
Salmeterol and formoterol are the two long acting beta 2 agonists available for asthma

Borderline Personality Disorder

 Cluster B personality disorder

 F>M

 Splitting

 Unstable mood and relationships

 Self-mutilation, ↑ suicide risk


schizoid personality disorder (D) are socially withdrawn, isolated, prefer to live alone and lead
solitary lifestyles. They are classically described as “loners” as they have very few close
interpersonal relationships.

Question: What is the medication of choice for the treatment of prolactinoma?

Answer: Cabergoline.

A pregnancy test is an appropriate first step in the evaluation of a woman


with amenorrhea and galactorrhea who is not taking medications. If the test is
negative, hyperprolactinemia should be suspected. Prolactin levels can be elevated because of
medications, pituitary adenoma, hypothyroidism, or mass lesion compromising normal hypothalamic
inhibition. Elevated prolactin levels inhibit the secretion and effect of gonadotropins.

Ciprofloxacin is an oral fluoroquinolone with adequate Pseudomonas aeruginosa coverage. Care


must be taken in children with ciprofloxacin because of concern over the development of
tendonopathy. In considering all skin and soft tissue infections PUNCTURE WOUNDS

Question: Why does the elderly population have an increased risk for digoxin toxicity?

Answer: Digoxin is a water-soluble and the volume of distribution of water-soluble


medications in the elderly is decreased.
As patients become older, there is a relative increase in body fat and a relative decrease in lean
body mass,

There is a reduction in glomerular filtration rate (B) in the kidneys that occurs with aging, which
causes decreased clearance of medications in the geriatric population, also increasing the risk of
drug toxicity. The absorption of drugs (A) is not affected with increasing age. There is a relative
decrease in lean body mass (C) as age increases.

Question: In what setting would a woman with positive GBS vaginal-rectal cultures not
require intrapartum antibiotic prophylaxis?

Answer: A planned cesarean delivery in the absence of labor or rupture of membranes.

Question: What is the recommended age for the administration of the Human
Papillomavirus Virus (HPV) vaccine?

Answer: The Human Papillomavirus vaccine is recommended to all males and females
aged 9-26 years
. The meningococcal vaccine (B) is a routine vaccination series given throughout childhood to
prevent meningitis. It is also recommended for college students up to 21 years of age who are living
in residence halls if they did not receive the vaccine at 16 year of age or older.

Women aged 50 to 74 years The USPSTF recommends biennial screening mammography for women aged 50 to 74 years.

For women aged 30 to 65 years who want to lengthen their screening interval, screening with a
combination of cytology and human papillomavirus (HPV) testing every 5 years is recommended

Management of diaper dermatitis involves skin care measures such as more frequent diaper
changing, air exposure, gentle cleansing, and use of topical barrier preparations. The most
common over-the-counter topical barriers contain petrolatum, zinc oxide, or both.

Question: What would thyroid function tests reveal in a patient with medullary thyroid
cancer?

Answer: Thyroid function tests are normal in patients with medullary thyroid cancer.
Multiple endocrine neoplasia type 2 is associated with medullary thyroid cancer, pheochromocytoma,
and primary parathyroid hyperplasia.

The production of calcitonin and carcinoembryonic antigen (CEA) is characteristic of medullary


thyroid cancer, and these are useful markers for diagnosis and follow-up. Patients typically present
with dysphagia, diarrhea, facial flushing, and a solitary thyroid nodule. Medullary thyroid cancer
is a neuroendocrine tumor of the parafollicular or C cells of the thyroid gland. The diagnosis of
medullary thyroid cancer is made after fine needle aspiration biopsy of the thyroid nodule.

Endometrial Cancer

 Patient will be a post-menopausal woman


 Complaining of abnormal vaginal bleeding

 Diagnosis is made by transvaginal ultrasound or endometrial biopsy

 Most common type is adenocarcinoma

 Treatment is total abdominal hysterectomy and bilateral salpingo-oophorectomy (TAH-


BSO
The most common cause of vaginal bleeding after menopause is atrophy of the vaginal mucosa or
endometrium

ho can’t tolerate endometrial biopsy should be referred for transvaginal ultrasound

Solar or actinic purpura (also known as senile purpura) is a common and benign condition of
extravasated blood in the dermis characterized by ecchymotic, purpuric patches on the forearms,
arms, or legs of older persons

Solar or actinic purpura (also known as senile purpura) is a common and benign condition of
extravasated blood in the dermis characterized by ecchymotic, purpuric patches on the forearms,
arms, or legs of older persons.

The condition usually follows minor trauma and commonly affects those who take aspirin or other
blood thinners such as coumadin or heparin.

Intravenous recombinant tissue plasminogen activator (rtPA) (C) is first line therapy for
adult patients presenting with an acute ischemic stroke. The therapeutic window for intravenous
rtPA is less than 4.5 hours from symptom onset. It is essential to administer intravenous rtPA as
soon as possible in the clinical setting of acute ischemic stroke. Treatment with intravenous rtPA
is associated with long term improvement in global disability, activities of daily living and
neurological deficits. Intracranial hemorrhage is an absolute contraindication and needs to be
ruled out with neuroimaging prior to administration of intravenous rtPA. The major risk of
intravenous rtPA treatment is intracranial hemorrhage.

Lumbar puncture (D) should be performed if there are symptoms suspicious of intracranial
infection, such as fever, altered mental status, headache or stiff neck. Lumbar puncture is also
indicated if there is suspicion of subarachnoid hemorrhage, such as in the setting of sudden
onset headache or trauma. CT of the head is required prior to lumbar puncture to rule out
hemorrhage or mass lesion in a patient with focal neurological deficits given concern for
cerebral herniation with increased intracranial pressure.

he rubella vaccine is a live vaccine, and the virus may cross the placenta and infect the fetus.
Therefore, it is contraindicated to administer the rubella vaccine (B) to pregnant women. In
the setting of a confirmed diagnosis of an acute rubella infection in a pregnant patient, the
patient should be counseled about maternal-fetal transmission with potentially devastating
effects on the fetus and offered pregnancy termination

A one-time screening for abdominal aortic aneurysm (AAA) with ultrasonography (A) is
recommended in men aged 65 to 75 years who have a smoking history
fecal occult blood testing is recommended annually, sigmoidoscopy is performed every 5 years or
colonoscopy is offered every 10 years

Pneumococcal vaccine is recommended for all adults aged 65 or older. Patients aged 2 years
or older who are immunocompromised or who have chronic diseases should also receive the
pneumococcal vaccine. Adults aged 65 or older should be revaccinated if they received their first
pneumococcal vaccine greater than five years ago or if they are considered high risk.

This patient has COPD, an underlying chronic disease, and therefore should receive the
pneumococcal vaccine today and should be revaccinated at age 65.

Placental Abruption

 Patient will be a pregnant women in her third trimester

 With a history of hypertension, trauma or cocaine use

 Complaining of painful vaginal bleeding

 Labs will show hypofibrinogenemia

 Question: What is the clinical condition when the placenta attaches to the
myometrium?

 Answer: Placenta accreta.


Placental abruption is the separation of the placenta from the uterine wall before delivery and is
considered an obstetric emergency. Placental abruption typically manifests as painful vaginal
bleeding and evidence of fetal distress in the third trimester. The fundus often is tender to palpation,
and pain occurs between contractions. Bleeding may be completely or partially concealed or may be
bright, dark, or intermixed with amniotic fluid. Common risk factors include abdominal trauma,
maternal hypertension, smoking, cocaine use, multiple gestation and previous abruption.

Question: What is the initial newborn screening test for cystic fibrosis?

Answer: Serum immunoreactive trypsinogen (IRT).

The test for chronic granulomatous disease is the nitroblue tetrazolium test (A). This provides
a simple and rapid determination of phagocyte NADPH oxidase activity. Common variable
immunodeficiency is a disorder involving immune dysfunction of B and T cells and dendritic cells
and is diagnosed with quantitative serum immunoglobulin levels (B). The clinical
manifestations of pediatric HIV infection are varied and often nonspecific (e.g.,
lymphadenopathy, oral candidiasis, failure to thrive). A serum HIV test (C) is warranted in
infants with perinatal HIV exposure.
Question: Which laboratory test related to bone metabolism should be performed in
patients with squamous cell carcinoma and breast cancer?

Answer: Parathyroid hormone-related peptide (PTHrP).

rine protein electrophoresis (UPEP) is used to identify the presence of the Bence Jones protein in
urine and bone marrow aspiration for increased plasma cells are all diagnostic and specific to
diagnose multiple myeloma.

Question: What readily available condiment will kill adult lice within 10 minutes of
application?

Answer: Mayonnaise.

Selenium sulfide shampoo (D) is used to treat fungal infections of the scalp and skin, however, is
ineffective in the treatment of head lice.

ermethrin is first-line therapy for treatment of pediculosis capitis (head


lice) in all populations, including pregnant and lactating women as well as
infants greater than or equal to 2 months of age. It has a 95% cure rate when
used correctly. Permethrin causes a respiratory paralysis of the adult louse
Acute intermittent
porphyria
Porphobilinogen
deaminase
Porphobilinogen,
δ-ALA,
coporphobilinogen
(urine)
Symptoms (5 P’s):
ƒ. Painful abdomen
ƒ. Port wine–colored urine
ƒ. Polyneuropathy
ƒ. Psychological disturbances
ƒ. Precipitated by drugs (eg, cytochrome P-450
inducers), alcohol, starvation
Treatment: glucose and heme, which inhibit
ALA synthase.

Pseuedodepression=look for B12 OR HYPOTHYROIDISM

Erythema Nodosum
 F >> M
 Causes: Streptococcus, sarcoid, fungal infections, IBD, TB, leukemia/lymphoma
 Medication causes: OCPs, sulfas
 Red tender nodules on shins

 Arthralgias
Gestational hypertension is defined by the new onset of hypertension (systolic blood
pressure ≥ 140 mm Hg or diastolic blood pressure ≥ 90 mm Hg) at ≥ 20 weeks of gestation
in the absence of proteinuria or signs of end-organ dysfunction. The blood pressure
readings should be documented on at least two occasions at least four hours apart. Gestational
hypertension is severe when systolic blood pressure is ≥ 160 mm Hg or diastolic blood
pressure is ≥ 110 mm Hg. If a patient has severe gestational hypertension that can be
controlled with oral medication, does not develop preeclampsia, and the fetus remains healthy,
delivery is recommended at 34 to 36 weeks.

Question: What is magnesium sulfate used for in pregnant patients with a blood
pressure ≥ 160/110 mm Hg?

Answer: Seizure prophylaxis

Delivery at 37 weeks (C) is recommended in women with frequent blood pressures ≥ 140/90
mm Hg and < 160/110 mm Hg, comorbidities, or other risk factors for adverse outcome.
Delivery at 38 to 39 weeks (D) is recommended for uncomplicated pregnancies with only
occasional blood pressures ≥ 140/90 mm Hg and < 160/110 mm Hg.

34 -36 is the answer

Delusional disorder is a primary psychotic disorder in which the most prominent symptom is
delusional beliefs, beliefs held with strong conviction despite superior evidence to the contrary.
Patients with delusional disorder may be high-functioning and coherent, but lack good
insight into their pathological experiences. In order for delusional disorder to be diagnosed,
the delusions must be present for at least one month. In addition, patients cannot have
prominent psychotic or accompanying symptoms characteristic of another psychiatric diagnosis.

Delusional disorder Fixed, persistent, false belief


system lasting > 1 month. Functioning otherwise not
impaired
(eg, a woman who genuinely believes she is married to a
celebrity when, in fact, she is not).
Can be shared by individuals in close relationships (folie à
deux).
Olfactory or tactile hallucinations related to the delusion may occur in patients with delusional
disorder;

Cardiac Tamponade

 Patient will be complaining of dyspnea and chest pain


 PE will show muffled heart sounds, JVD, hypotension (Beck’s
triad), pulsus paradoxus

 ECG will show low voltage QRS, electrical alterans

 Echocardiography will show a diastolic collapse of RV

 Treatment is pericardiocentesis

A sentinel lymph node biopsy (D) may be used for staging after a melanoma is diagnosed.

A complete full-thickness excisional biopsy of suspicious lesions with 1 to 3 mm margin of


normal skin and part of the subcutaneous fat should be performed whenever possible.

In afebrile infants one to four months of age with community-acquired pneumonia, the
most likely bacterial pathogen is Chlamydia trachomatis. Neonatal C. trachomatis pneumonia
presents between 4 and 12 weeks of age and is asymptomatic prior to presentation. Infants
are usually afebrile, have a staccato cough, and many will also have conjunctivitis. The gold
standard for diagnosis is with a culture. Initial therapy for chlamydial pneumonia is based on a
presumptive diagnosis until diagnostic test results are available. Prevention of neonatal C.
trachomatis infection is achieved by treating pregnant women with genital C. trachomatis
because the infection is passed from the mother to the infant in the birth canal.

Question: Which antibiotics are used to treat hospital-acquired aspiration pneumonia?

Answer: Imipenem, meropenem, or piperacillin-tazobactam.

Most patients with aspiration pneumonia acquired in the community have a mixed bacterial
infection that includes strict anaerobes and facultative anaerobes such as oral streptococci.
Intravenous ampicillin-sulbactam is first-line therapy.

Question: What drug commonly used for maintenance therapy of gout is avoided
during an acute gout attack?

Answer: Allopurinol. Except in diuretic-induced gout.

Loop diuretics

Ototoxicity, Hypokalemia, Dehydration,


Allergy (sulfa)/metabolic Alkalosis, Nephritis
(interstitial), Gout.

Furosemide is the most likely medication to be contributing to this patient’s acute attack of gouty
arthritis. Hyperuricemia is a relatively common finding in patients treated with a loop or thiazide
diuretic and may, over a period of time, lead to gouty arthritis. Diuretics may increase the relative risk
of gout by nearly 80%. Loop and thiazide diuretics decrease urate excretion by increasing net urate
reabsorption; this can occur either by enhanced reabsorption or by reduced secretion.

Summary of Recommendation and Evidence. The USPSTF recommends annualscreening for


lung cancer with low-dose computed tomography (LDCT) in adults aged 55 to 80 years who
have a 30 pack-year smoking history and currently smoke or have quit within the past 15 years.
USMLE

Screening should be discontinued once a person has not smoked for 15 years or develops a health
problem that substantially limits life expectancy or the ability or willingness to have lung surgery.

Nasopharyngeal carcinoma (D) is associated with Epstein-Barr virus (EBV) infection.

Question: Which infectious agent is associated with Burkitt lymphoma?

Answer: Epstein-Barr virus.


H. pylori infection. H. pylori is a gram-negative, urease-producing, flagellated bacterium that lives on
the mucosal lining of the stomach. It is believed that bacterial production of cytotoxins and proteases
leads to mucosal injury and clinical symptoms of gastritis and peptic ulcer disease.

Angiotensin-converting enzyme (ACE) inhibitors are the leading cause of drug-induced


angioedema. Patients most commonly present with swelling of the lips, tongue, or face.

The primary acute treatment of ACE inhibitor-induced angioedema is supportive care until the
angioedema resolves.

US Preventive Services Task Force recommends that women of childbearing potential take a daily
supplement containing 0.4 to 0.8 mg (400 to 800 µg) of folic acid. This recommendation is due to
the fact that the neural tube closes very early in gestation before many women know they are
pregnant.

Women who are planning or The USPSTF recommends that all women who are planning or capable of pregnancy take a
capable of pregnancy daily supplement containing 0.4 to 0.8 mg (400 to 800 µg) of folic acid.

Question: What is the first line therapy for treatment of bacterial pneumonia in infants
and preschool aged children?

Answer: High-dose amoxicillin (80-90 mg/kg/day).

A one-month-old well-appearing girl with lobar pneumonia (B) should be admitted based on her age.
A seven-month-old girl with an oxygen saturation of 90% on room air (C) is hypoxic and therefore
should be admitted. A nine-month-old boy with multi-lobar pneumonia (A) should be admitted based
on the multi-lobar nature of his pneumonia.
S CA19-9 (B) is elevated in pancreatic cancer, gallbladder cancer, bile duct cancer, and gastric
cancer. CA-125 (A) is elevated in ovarian cancer and carcinoembryonic antigen (C) is elevated in
colorectal cancer and breast cancer. None of these tumor markers are elevated in testicular cancer.
erum lactate dehydrogenase elevations reflect the rapid turnover of malignant cells

Meniere's Disease

 Patient will be complaining of episodic low-frequency hearing loss, tinnitus with aural
(ear) fullness, and vertigo lasting one to eight hours

 Diagnosis is made clinically

 Most commonly caused by too much inner ear endolymph and increased pressure
within the inner ear

 Treatment is low salt diet, diuretics (HCTZ + triamterene)

A scopolamine patch (D) is useful for treatment of a single episode of vertigo, but not long-term
management for Meniere's.

Excess pressure accumulation in the endolymph can cause a tetrad of symptoms


including fluctuating hearing loss, episodic vertigo, tinnitus, and aural fullness.

Scabies

 Diagnosis is made by microscopic visualization

 Treatment is permethrin 5%
Ivermectin (A) should not be used during pregnancy and is not safe to use in children less than five
years.

Lindane 1% cream (B) is a pregnancy category C drug and has been shown to be absorbed
systemically in studies

Question: What is the treatment for a febrile seizure that continues for more than 5
minutes?

Answer: Intravenous benzodiazepines are effective in aborting seizure in many cases.

Simple febrile seizures are the most common and are characterized by seizures that last
less than 15 minutes, have no focal features, and occur once in a 24-hour period.

Children who experience a febrile seizure have a 30% chance of at least one recurrence.

Question: How are night splints beneficial in plantar fasciitis?

Answer: Night splints maintain a neutral 90° foot-leg angle and provide constant passive stretching of the
Achilles tendon and plantar fascia.
heel pain that is exacerbated by standing on her feet all day at work. She complains that the
pain is worse in the morning upon getting out of bed but subsides within 30-45 minutes. On
physical exam, pain is elicited by dorsiflexion of the toes.

repetitive microtrauma. Risk factors include obesity, occupations requiring prolonged


standing and weight bearing, and heel spurs. Most patients report that the pain usually
is most severe during the first few steps after prolonged inactivity, such as sleeping or
sitting but usually resolves upon warming up within 45 minutes. Patients who are
generally standing on their feet all day report that the symptoms may actually worsen by
the end of the day. Physical exam maneuvers that may reproduce the pain of plantar
fasciitis include passive dorsiflexion of the toes, and having the patient stand on the
tiptoes and toe-walk. Radiographs typically are not necessary for diagnosing plantar
fasciitis. However, to rule out a bony tumor or fracture, always consider obtaining at
least a plain radiograph before administering a corticosteroid injection. Treatment is
directed at decreasing the inflammation. These treatments include icing, nonsteroidal
anti-inflammatory drugs, rest and activity modification, corticosteroids, botulinum toxin
type A, splinting, shoe modifications, and orthoses.

Excising the entire lesion with 5 mm margins (C) or 1-3cm margins (A) is not indicated for a
biopsy as these wide margins can cause cosmetic defects. If the histology returns indicating
melanoma then surgical excision with narrow margins is the preferred treatment of choice

Elliptical excision of the suspicious lesion with a small margin (approximately 1-3 mm) of normal-
appearing skin should be performed and the lesion should be sent to pathology to assess for tumor
thickness, ulceration, histologic level of invasion and the mitotic rate.

TTP/HUS
 HUS:
 E. coli 0157:H7
 Autoimmune hemolysis, Renal failure, Thrombocytopenia (ART)
 Bloody diarrhea
 Avoid ABX
 TTP:
 ART + Fever + Neurologic abnormalities (FAT RN)
 Normal coagulation studies

 Plasmapheresis
 Hemorrhagic colitis that is usually
 self-limited, but has been
 associated with hemolytic-uremic
 syndrome (HUS) and thrombotic
 thrombocytopenic purpura (TTP);
 supportive treatment, do not give
 antibiotics (induces the release of
 Shiga toxin, increasing the risk of
 HUS/TTP)


 Blepharitis
 a. Inflammation of the eyelid; often associated with infection with Staphylococcus
 spp.
 b. Usually diagnosed by careful examination of the eyelid margins, which are red
 and often swollen with crusting that sticks to the lashes
 c. Treat with lid scrubs and warm compresses. Give topical antibiotics for severe
 cases

Blepharitis is a common inflammatory condition of the eyelids which can become chronic. It is
diagnosed clinically based on symptoms and exam findings. The lids may appear
erythematous and irritated. Often times there is associated crusting along the lid margins or
lashes. Conjunctival injection may also be present making it difficult to distinguish from acute
conjunctivitis. Patients may complain of a vast array of symptoms including a gritty or foreign
body sensation, red eye, excessive tearing, itchy, red swollen eyelids, crusting of the lids and
lashes in the morning, blurred vision or even light sensitivity. Treatment should center on patient
education on how to minimize exacerbations as the condition can become chronic. Patients
should be encouraged to practice appropriate and routine lid hygiene making sure to cleanse
around the eye area with a mild soap or diluted baby shampoo regularly. Warm compresses
should be applied to the eyes two to four times per day during acute exacerbations. Antibiotics
can also help to improve symptoms as well. Ointment preparations such as erythromycin are
typically recommended and should be applied directly to the eyelid margin up to four times per
day.

fibroadenoma if a young
woman presents with a round, movable mass which changes in size
over the course
of the menstrual cycle.

Other breast masses which may occur at any age include cysts (benign if not
bloody after aspiration and does not recur

fibrocystic
changes (bilateral breast pain caused by cyclic
hormonal stimulation).

Bipolar disorder

 Lifelong, extreme mood swings

 Mania, hypomania

 Inflated self esteem

 Decreased need for sleep

 Pressured speech
 Flight of ideas

 Excessive pleasurable activity

 Question: What common analgesics can alter lithium levels?

 Answer: NSAIDs, with the exception of aspirin.


Thyroid function studies are routinely monitored in patients on lithium therapy. Lithium is a
mainstay of treatment for bipolar disorder and may reduce the risk of suicide.
Prior to beginning lithium, clinicians should obtain a urinalysis, blood urea nitrogen, creatinine,
thyroid function studies, calcium, pregnancy test for women of childbearing potential, and an ECG
for patients over age 40.
Thyroid studies are also monitored in the setting of amiodarone therapy.

A diagnosis of uterine atony can be made if the uterus does not become firm during the third
stage of labor with uterine massage and oxytocin

Post–Lumbar Puncture Headache

 Due to persistent CSF leak

 24–48 hours after LP

 Worsened by sitting/standing, improved by laying flat

 Preventive measures: small caliber needle, blunt needle, stylet replacement


before needle

 Management: hydration, IV caffeine, epidural blood patch

Question: What medications increase the likelihood of patient experiencing severe or fatal anaphylaxis?

Answer: Beta-adrenergic blockers, angiotensin-converting enzyme (ACE) inhibitors, and alpha-adrenergic


blockers.

Second-generation H1 antihistamines, such as oral cetirizine (C), can help to relieve itching
and urticaria, however, antihistamines do not relieve upper or lower airway obstruction,
hypotension or shock, or inhibit mediator release from mast cells and basophils. Intramuscular
epinephrine is preferred over subcutaneous epinephrine (D), because intramuscular injections
provide a more rapid increase in the plasma and tissue concentrations of epinephrine.

However, women with gestational diabetes mellitus are at increased risk of developing
diabetes and should have their fasting glucose levels monitored for 24 to 72 hours after
delivery to check for overt diabetes (fasting glucose > 126 mg/dL or random glucose ≥ 200
mg/dL). If overt diabetes is not diagnosed postpartum, women with gestational diabetes mellitus
should be screened or tested for diabetes with a two-hour 75-gram oral glucose tolerance
test 4 to 12 weeks after delivery to establish glucose status
Radioiodine therapy (C) for pregnant women with hyperthyroidism is absolutely
contraindicated. For women with symptomatic, moderate-to-severe, overt hyperthyroidism who
cannot tolerate thioamides because of allergy or agranulocytosis, thyroidectomy (D) in the
second trimester may be necessary.

Genital Herpes Simplex

 Patient will be complaining of a painful genital rash

 PE will show grouped erythematous, shallow, cluster of vesicles


and lymphadenopathy

 Labs will show multinucleated giant cells on Tzanck smear

 Diagnosis is made clinically. Gold standard is tissue culture with polymerase chain
reaction (PCR)

 Most commonly caused by herpes simplex virus (HSV) type 2

 Treatment is acyclovir

Penicillin (C) is first-line treatment of syphilis, which causes a painless chancre on the penis.
Ottawa Knee Rule (OKR) — The OKR states that radiographs of the
knee should be obtained after acute injury ONLY for patients who meet one or more of the
following criteria [26]:

●Age ≥55 years

●Isolated tenderness of patella (with no other bony tenderness of the knee)

●Tenderness at the head of the fibula

●Inability to flex the knee to 90 degrees

●Inability to bear weight both immediately and in the emergency department for four steps,
regardless of limp (ie, unable to transfer weight onto each lower limb two times)

In a meta-analysis, the OKR was almost 100 percent sensitive for identifying patients who have
knee fractures (99 percent, 95% CI 93-100 percent) and 49 percent specific (95% CI 43-51
percent) [27].

Iliotibial band syndrome is an overuse injury of the lateral knee that occurs primarily in
runners and cyclists. Pain develops where the iliotibial band courses over the lateral
femoral epicondyle proximal to the lateral knee joint line. The diagnosis is made clinically
based upon a suggestive history and characteristic examination findings. A positive Noble
compression test is the best test to confirm iliotibial band syndrome. Acute treatment involves
up to two weeks of rest from activities that reproduce the pain, the application of ice to the
lateral knee, and taking anti-inflammatories to control the pain. Referral to physical therapy
for the correction of strength and mobility deficits is appropriate in this subacute phase of
treatment.

4. De Quervain disease
a. Pain at the radial aspect of the wrist (especially with pinch gripping) in
region
of radial styloid; common for pain to radiate to elbow or into thumb.
b. Due to inflammation of the abductor pollicis longus and extensor pollicis
brevis
tendons.
c. Positive Finkelstein test—have the patient clench the thumb under the
other
fingers when making a fist. Then ulnarly deviate the wrist. The test is
positive
if pain is reproduced.
d. Treatment is thumb spica splint and NSAIDs. Local cortisone
injections can be
helpful. Surgery done if conservative measures fail and is usually
effective.
Symptoms of carpal tunnel syndrome affect most patients at night and when the wrist is overly
flexed or extended.

A positive Phalen sign is pain or paresthesia in the median distribution following one minute of
wrist flexion.

Alopecia Areata

 Patient with a history of an autoimmune disorder

 Complaining of hair loss

 PE will show patches of smooth, non-scarring hair loss with patches of smaller
hairs termed “exclamation hairs”

 Most common cause is autoimmune

 Treatment is intralesional corticosteroids

 Severity can range from intermittent small patches on the scalp to loss of all hair on the scalp
or body. The physiology of this condition is unknown, but it is believed to be a T-cell mediated
autoimmune condition.

Patients with alopecia areata will present with smooth,circular discrete areas of hair loss, mos
First-line treatment for mild cases is intralesional or topical corticosteroids. More severe
cases should be referred to dermatology.

 t commonly on the scalp.

 Question: True or false: patients with alopecia areata can experience spontaneous regrowth of
their hair?

 Answer: True.

Telogen effluvium is an acute form of alopecia caused by medications, hormonal stress or


metabolic stress. Clinical presentation is a generalized reduction in scalp hair density (B).
Androgenetic alopecia, commonly called male pattern baldness, is a permanent condition that
presents with thinning hair at the temporal, frontal and vertex regions of the scalp

Pharyngitis

 Patient will be complaining of a sore throat

 PE will show fever, cervical lymphadenopathy, tonsillar exudate, and no cough


 Most commonly caused by viral > bacteria (GAS, S. Pygoenes)

 Centor Score: Estimates probability that pharyngitis is streptococcal

 Treatment is:

 Viral - supportive

 Bacterial - PCN (first-line)

 PCN allergic:

 Cephalexin, cefadroxil (avoid in individuals with immediate type hypersensitivity to


penicillin)

 Clindamycin

 Azithromycin, clarithromycin (resistance of group A strep to these agents is well-known


and varies geographically and temporally)

References:

“Strep throat,” caused by Group A Beta-hemolytic streptococcus (Streptococcus pyogenes or


GAS), is a common etiology of acute pharyngitis especially in children ages 5 to 15. It is
characterized by inflammation of the pharynx or tonsils (tonsillar exudates) associated with
symptoms of fever, malaise and sore throat, as well as the absence of other URI symptoms such
as nasal congestion and cough. Cervical lymphadenopathy is often found on exam, as is a
whitish exudate over the pharynx and tonsils. A rapid streptococcal antigen test is recommended
in order to determine if treatment with antibiotics is warranted, as other conditions which do not
require antibiotic treatment may mimic streptococcal pharyngitis. A throat culture to rule-out GAS
infection is recommended in children if rapid antigen testing is negative (90% sensitivity), in order to
limit transmission and prevent complications such as rheumatic fever. Other complications of strep
throat may include acute glomerulonephritis, peritonsillar abscess, bacteremia, sinusitis and
pneumonia. Penicillin-based antibiotics (benzathine penicillin IM or oral penicillin VK) are the
treatment of choice. For penicillin allergic patients, azithromycin is an alternative.
Mononucleosis (A) is an infection caused by Epstein-Barr virus and is most common in 15 to 24
year-olds. Patients usually experience a several day prodrome of fever, chills, malaise and anorexia
followed by the onset of throat pain, fever and lymphadenopathy. The diagnosis is supported by
heterophile (Monospot) antibody testing. Treatment is supportive and the illness is generally self-
limited, though some patients may experience residual symptoms of malaise and fatigue for months
following initial diagnosis.

inusitis (B) refers to inflammation of the mucous membranes lining the paranasal sinuses. Patients
present complaining of cough, facial tightness, headache, reduced ability to smell and nasal
congestion with purulent nasal and postnasal discharge lasting 7-10 days. Sinusitis is usually
caused by a virus, leading to decreased clearance of secretions within the mucosa and entrapment
of bacteria which may lead to secondary bacterial infection. The three most common bacterial
agents involved in sinusitis are Haemophilus influenzae, Streptococcus pneumoniae and Moraxella
catarrhalis. Treatment of viral sinusitis is supportive; antibiotics are indicated only if the clinical
course strongly suggests bacterial secondary infection.

Question: What is the only US FDA-approved inhaled glucocorticoid for children


younger than four years of age?

Answer: Budesonide.
If asthma control is not achieved with low-dose inhaled glucocorticoids, a trial of a high-dose
inhaled glucocorticoid (B) is warranted or the addition of a second controller agent should be
added to the patient's daily regimen.

A 10-year-old girl presents to your clinic for asthma management. She was diagnosed with mild
intermittent asthma one year ago and was given a short-acting beta-agonist rescue inhaler at
that time. Her mother reports that she has been using the inhaler four times per week over the
past month and wakes up almost every night with a dry cough. Which of the following is the
most appropriate next step in management?

Patients who experience intermittent symptoms of asthma should use a short-acting,


inhaled beta-2-selective adrenergic agonist which is considered “step 1 therapy” for relief of
acute symptoms as well as for prevention of symptoms caused by exercise or trigger exposure.
For patients with mild, persistent asthma, low-dose, daily inhaled glucocorticoids are
recommended as the first-line controller therapy. Glucocorticoids work by inhibiting steps in
the cascade of the inflammatory response, reduceding bronchial hyperresponsiveness,
preventing the asthmatic response, and enhancing lung function. Inhaled glucocorticoids are
delivered directly to the airways at a lower dose than what would be required if taken
systemically, and have minimal side effects.

Hyperkalemia

 Patient with a history of renal failure, DKA, rhabdomyolysis, tumor lysis


 Complaining of lethargy, weakness, paralysis

 PE will show bradycardia, hypotension, cardiac dysrhythmia

 ECG will show peaked T waves, prolonged PR, wide QRS

 Treatment is calcium gluconate, insulin, albuterol, kayexalate, bicarbonate

 Question: What is the treatment for hyperkalemia resulting in metabolic


acidosis?

 Answer: Sodium bicarbonate.

Hyperkalemia is defined as a serum potassium concentration greater than 5.5 mEq/L in adults.
As the potassium concentration increases, the patient is at increased risk of a lethal dysrhythmia.
In patients with severe hyperkalemia, treatment focuses on immediate stabilization of the
myocardial cell membrane.

Tinnitus is a perception of sound in the absence of an external source.

Tricyclic antidepressants such as nortriptyline may cause QT prolongation resulting in


torsades de pointes.

procainamide,quinidine, flecanide, propafenone

Question: Cotton wool spots and flame hemorrhages are common physical findings on
fundoscopic exam in which disorder?

Answer: Diabetic retinopathy.

Macular Degeneration

 Patient will be older

 Complaining of bilateral, gradual central field vision loss

 PE will show

 Dry macular degeneration (85% of cases): Atrophic changes and yellow retinal
deposits (Drusen spots)

 Wet macular degeneration: Vascular changes

 Diagnosis is made by Amsler grid

 Most common cause of blindness in the elderly


Central retinal artery occlusion (C) causes sudden, acute, and painless loss of vision in one
eye. Fundoscopic exam will show a red lesion, called a "cherry red spot," with surrounding pale
retina.

Age related macular degeneration is the most common cause of blindness in older
Americans.
Cystitis

 Patient will be complaining of low-grade fever, increased urinary frequency, dysuria,


and suprapubic or abdominal pain.

 Labs will show positive leukocyte esterase and nitrites

 Definitive diagnosis is made by urine culture

 Most commonly caused by Escherichia coli

 Treatment is:

 Acute uncomplicated cystitis: TMP-SMX, nitrofurantoin, or fluoroquinolone


for 3-5 days

 Acute uncomplicated cystitis with comorbid conditions: TMP-SMX,


nitrofurantoin, or fluoroquinolone for 7 days

 Pregnancy - asymptomatic bacteruria - treat

 Cephalosporins, nitrofurantoin (during first trimester - only when needed)

 Complications: ↑ Risk of preterm birth, low birth weight, perinatal mortality

Question: Which class of antibiotics are contraindicated in patients with risk factors for
torsades de pointes?

Answer: Fluoroquinolones.
Euthyroid sick syndrome (ESS) is a state of adaptation or dysregulation of thyrotropic feedback
control[1] wherein the levels of T3 and/or T4 are abnormal, but the thyroid gland does not appear to
be dysfunctional.
This condition is often seen in starvation, critical illness, or patients in the intensive care unit. Similar
endocrine phenotypes are observed in fetal life and in hibernating mammals. [2] The most common
hormone pattern in NTIS is low total and free T3, elevated rT3, and normal T4 and TSH levels,
although T4 and TSH suppression may occur in more severe or chronic illness. [3]

Associated with
cancer
cardiac disease (MI, CHF)
CRF
sepsis
Evaluation
Serology
↓ total serum T4/T3
↓ serum TSH
↑ reverse T3

Euthyroid sick syndrome can be described as abnormal findings on thyroid function


tests that occur in the setting of a nonthyroidal illness, without preexisting hypothalamic-
pituitary and thyroid gland dysfunction. This is most commonly diagnosed in a
hospitalized patient and after recovery from the underlying illness. The thyroid function test
abnormalities subsequently resolve. The most prominent alterations are low serum
triiodothyronine (T3). Thyroid-stimulating hormone (TSH), thyroxine (T4), and free T4 (FT4) are
also affected in variable degrees based on the severity and duration of the nonthyroidal illness.
As the severity of the underlying illness increases, both serum T3 and T4 levels drop and
gradually normalize as the patient recovers.

Subclinical hyperthyroidism (D) is defined as an abnormally low serum thyroid-stimulating


hormone level (TSH) and normal levels of free thyroxine (T4) and triiodothyronine (T3).
Subclinical hyperthyroidism has the potential to convert to overt hyperthyroidism.

Question: What are three common causes of delirium?

Answer: Infections, medications and metabolic derangement.

The use of benzodiazepines should be avoided except in cases of alcohol or sedative-hypnotic


withdrawal

Physical restraints (D) and bed rails actually increase the risk of injury in patients with
delirium. Feeding tubes (A) and foley catheters (B) are the first devices to get yanked out by a
patient and cause injury in acute delirium; therefore they should definitely not be inserted during
delirium.
Barrett’s Esophagus

 Patient with a history of chronic reflux

 Diagnosis is made by upper endoscopy, biopsy

 Biopsy will show squamous to columnar epithelium and proximal shift in the
squamocolumnar junction

 Treatment is proton pump inhibitors

 Comments: Increased risk for adenocarcinoma

 Question: What are the three medications used in triple therapy for Helicobacter
pylori eradication?

 Answer: Omeprazole, clarithromycin, and amoxicillin.

In Barrett's esophagus, metaplastic columnar epithelium replaces the stratified squamous


epithelium that normally lines the distal esophagus. The metaplastic epithelium is acquired as a
consequence of chronic gastroesophageal reflux disease and predisposes to the
development of malignancy. All patients with Barrett's esophagus are indefinitely treated with
a proton pump inhibitor (PPI) since aggressive antireflux therapy may prevent cancer
development. The goal of surveillance is to improve outcomes by detecting dysplasia or
esophageal adenocarcinoma early enough to offer effective treatment. Current guidelines
suggest surveillance for most patients with Barrett's esophagus. If the initial biopsies show no
dysplasia, surveillance is recommended every three to five years.
Endoscopic eradication therapy (A) is recommended for patients who are confirmed to have
high-grade dysplasia or intramucosal carcinoma, and who have no evidence of submucosal
invasion in their resected specimens. Esophagectomy (C) is the only therapy for high-grade
dysplasia that removes all of the neoplastic epithelium along with any occult malignancy and
regional lymph nodes. If the initial biopsies are indefinite for dysplasia, medical antireflux therapy
with a proton pump inhibitor twice daily followed by a repeat endoscopy with biopsy after 2-3
months of treatment (D).

Question: What electrolyte abnormalities are most commonly associated with 21-
hydroxylase deficiency?

Answer: Hyponatremia and hyperkalemia.{DUE TO LACK OF ALDOSTERONE}

Due to the risk of adrenal crisis, any infant presenting with atypical genitalia should have a
rapid and extensive evaluation for congenital adrenal hyperplasia. Cortisol and
adrenocorticotropic hormone (ACTH) levels (B) are be helpful in identifying the diagnosis
of ACTH deficiency and should be measured after 17-hydroxyprogesterone

Atypical genitalia in females is most commonly caused by congenital adrenal


hyperplasia with 21-hydroxylase deficiency, which is a autosomal recessive disorder.
Complications of this condition include adrenal insufficiency which may occur within weeks
of birth. Patients with 21-hydroxylase deficiency have an elevated serum concentration of
17-hydroxyprogesterone. Neonatal screening for 21-hydroxylase deficiency is routinely
performed with a measurement of 17-hydroxyprogesteron
Question: What are the first-line therapies for tinea versicolor?

Answer: Topical antifungal medications including topical selenium sulfide and topical
zinc pyrithione.
Question: What is the mainstay of pharmacologic therapy for women with polycystic
ovarian syndrome for managing hyperandrogenism and menstrual dysfunction?

Answer: Oral contraceptives.

Women with polycystic ovary syndrome may present with oligomenorrhea,


hyperandrogenism, anovulatory infertility, and have metabolic risk factors such as obesity,
insulin resistance, dyslipidemia, and impaired glucose tolerance. While weight loss is the
first-line intervention used to restore ovulatory cycles and improve metabolic risk, many
women with polycystic ovarian syndrome rely on pharmacologic therapies to manage their
symptoms. Medications should always be assessed prior to conception. Statins are
contraindicated in pregnancy and should be discontinued prior to conception.

Question: Auer rods can be found in the blast cells of which leukemia?

Answer: Acute myelogenous leukemia.

A 5-year-old boy and his mother present to the clinic with a two-week history of fever and
fatigue. She states that he has seemed more tired and not himself for the past six weeks. He
has been less active, preferring to sit or to play indoors instead of going outside. He often wakes
up tired in the morning, even after sleeping 10 hours or more. He also has been complaining of
headaches. Two weeks ago he started running a mild fever which did not respond to ibuprofen,
acetaminophen, room temperature baths, or applying cool cloths. His breathing is sometimes
rapid, he has decreased appetite, and he has lost six pounds in the past month. She denies any
vomiting, diarrhea, urinary changes, abdominal pain, chest pain, runny nose, or cough. His past
medical history is unremarkable and his family history is negative for immunosuppressive
disorders or cancer. Vital signs include T 100.8°F, BP 105/75 mm Hg, HR 120 bpm, and RR
30/minute. Physical examination is significant for pallor, tachypnea, tachycardia, and non-pitting
lower extremity edema. Oropharynx is pink and moist and neck is supple without cervical
lymphadenopathy. Chest is clear to auscultation and cardiac exam reveals normal rhythm
without murmurs, rubs, or gallops. The abdomen is soft, flat, and nontender, without masses or
hepatosplenomegaly. Laboratory tests reveal a WBC count of 22.0 x 10 /L, hemoglobin of 11.5
9

g/dL, and platelet count of 156 x 10 /L. Which of the following signs or symptoms warrant a
9

peripheral blood smear when combined with the results of the patient’s complete blood count?

Pneumococcal polysaccharide vaccine 23 only (D) is recommended for adults who are < 65 years of age
who are current cigarette smokers, have chronic heart, lung, or liver disease, diabetes mellitus, or
alcoholism. Pneumococcal conjugate vaccine 13 only (B) is recommended for infants or toddlers < 2
years of age.

According to the Centers for Disease Control and Prevention (CDC) Advisory Committee on
Immunization Practices (ACIP), a dose of pneumococcal conjugate vaccine 13 should be
given to adults 65 years or older followed by a dose of pneumococcal polysaccharide
vaccine 23 at least one year later.

Short courses of systemic corticosteroids increase the time to subsequent exacerbations.in


COPD EXACERBATION

Question: What complication can arise from the abrupt cessation of high-dose
steroids?

Answer: Adrenal insufficiency.


The use of oral steroids in persons with chronic stable COPD is widely discouraged, given their
adverse effects, which include hypertension, glucose intolerance, osteoporosis, fractures, and
cataracts. These patients should receive inhaled regimens.

Question: What is the treatment for growing pains?

Answer: The treatment is reassurance, rest and NSAIDS.

Growing pains is a diagnosis of exclusion in children and often times presents as bilateral
leg pain after physical activity. It is a disease of unknown etiology however it is hypothesized
that it is due to overuse on an immaturely developed musculoskeletal system. Growing pains are
benign and usually resolve within a year or two of onset. They must be distinguished from
pathologic causes of pain that require additional therapy. It most commonly affects active children
2-6 years of age and in the pre-teen years. Pain is bilateral and can be localized to the thigh, calf
or ankles. Growing pain occurs primarily at night and resolves during the day. There are no
constitutional symptoms present in growing pain and radiographic studies are not necessary.

Ewing sarcoma (A) is a rare bone tumor that occurs in the long bones of the legs and arms, or
bones in the chest, trunk, pelvis, back, or head. A palpable mass and bone pain are seen on
clinical examination. Approximately half of all patients are aged 10-20 years at the time of first
diagnosis. Osteomyelitis (D) is inflammation of the bone caused by an infecting organism. The
major cause of bone infections is Staphylococcus aureus. Systemic symptoms are often seen
and erythema and warmth are usually present on physical exam. Juvenile rheumatoid arthritis
(C) presents with morning stiffness for at least 6 weeks duration that coincide with fever spikes
and dermatologic rashes.
Pertussis ("Whooping Cough")

 Patient with a history of nasal congestion, cough, and low-grade fever

 Complaining of “rapid fire,” repetitive coughing followed by an inspiratory “whoop"


and post-tussive emesis

 Most commonly caused by Bordetella pertussis

 Treatment is a macrolide – azithromycin

 Current recommendations in the United States consist of administering five doses of the
diphtheria and tetanus toxoids and acellular pertussis (DTaP) vaccine to children before seven
years of age, and administering a tetanus toxoid, reduced diphtheria toxoid, and acellular
pertussis (Tdap) booster between 11 and 18 years of age.
Tetralogy of Fallot

 Patient with a history of episodes of cyanosis (tet spells) and squatting for relief

 PE will show pulmonic stenosis, right ventricular hypertrophy, overriding aorta,


and VSD

 CXR will show "boot-shaped" heart

 Comments: Most common cyanotic congenital heart disease


 Mnemonic: PROVe:: Pulmonic stenosis, Right ventricular hypertrophy, Overriding
aorta, VSD

IMPET
IGO
 Impetigo is a contagious superficial bacterial infection which commonly presents
on the face or extremities with lesions that progress from papules to vesicles,
pustules, and crusts. The infection is most frequently observed in children and is
spread among individuals in close contact. Staphylococcus aureus is the most
common cause of impetigo. Impetigo may be followed by poststreptococcal
glomerulonephritis or rheumatic fever. Treatment with topical therapy, including
mupirocin or retapamulin, rather than oral therapy, is preferable in cases with a small
number of lesions.

 This patient is suffering from Alzheimer’s disease. It is a chronic neurodegenerative


disease that usually starts slowly and gets worse over time. The most common symptom is
difficulty in remembering recent events. After memory loss occurs, patients may also
experience language disorders and impairment in their visuospatial skills and executive
functions. It is the most common form of dementia, affecting nearly one-half of Americans
older than 85 years. Amyloid plaque accumulation, neurofibrillary tau tangles, and depletion of
acetylcholine are among the pathologic manifestations of Alzheimer
disease. Acetylcholinesterase inhibitors are first-line medications for the treatment of
Alzheimer’s disease, and are associated with mild improvements in cognitive function,
behavior, and activities of daily living. Memantine can also be used in combination with
acetylcholinesterase inhibitors. Age is the greatest risk factor for developing Alzheimer’s
dementia. The incidence increases greatly after 85 years of age.
Tetralogy of Fallot

 Patient with a history of episodes of cyanosis (tet spells) and squatting for relief

 PE will show pulmonic stenosis, right ventricular hypertrophy, overriding aorta,


and VSD

 CXR will show "boot-shaped" heart

 Comments: Most common cyanotic congenital heart disease

 Mnemonic: PROVe:: Pulmonic stenosis, Right ventricular hypertrophy, Overriding


aorta, VSD

 Question: What position should a patient be placed in in order to reduce the severity of
symptoms during a ‘”tet spell?”

 Answer: Squatting or knees drawn to chest, which increases systemic vascular resistance and
allows for a temporary reversal of the shunt.

Cyanosis is usually not the presenting sign for coarctation of the aorta (B) since the defect
involves a narrowed aorta that does not result in right-to-left shunt.

Tetralogy of Fallot is a congenital heart defect that classically involves four characteristic
features: ventricular septal defect (VSD), right ventricular hypertrophy (RVH), pulmonary
stenosis, and an overriding aorta. It is one of the most common cyanotic heart defects and a
common cause of “blue baby syndrome.” Tetralogy of Fallot causes a right-to-left shunt,
resulting in severe cyanotic symptoms. This right-to-left shunt results in low oxygenation of blood
due to the mixing of oxygenated and deoxygenated blood in the left ventricle via the ventricular
septal defect (VSD). The low oxygenated blood then preferentially passes through the aorta and
is delivered systemically due to the obstruction of flow through the stenotic pulmonary valve. The
primary symptom is low blood oxygen saturation with or without cyanosis from birth or developing
in the first year of life. Children with Tetralogy of Fallot may develop “tet spells” from a rapid drop
in the amount of oxygen in the blood. “Tet spells” manifests as a sudden development of deep
blue skin, nails, and lips after crying, feeding, crawling or upon awakening as well as rapid and
difficult breathing. On physical exam, a harsh systolic ejection murmur is heard over the pulmonic
area and left sternal border secondary to the stenotic pulmonary valve. Chest X-ray classically
depicts a “boot-shaped” heart and ECG reveals right axis deviation due to the right ventricular
hypertrophy. The definitive diagnosis of congenital heart defects are diagnosed via
echocardiogram.

Question: What is the definition of a hypomanic episode?

Answer: Abnormally and persistently elevated, expansive, or irritable mood that lasts
four days or more, with three or more of the symptoms of mania, but without psychotic
symptoms or serious impairment in normal functioning.

The patient’s symptoms of erratic behavior, talkativeness, racing thoughts, and little sleep for
three days indicate that he most likely is having a manic episode. A manic episode is
defined as a period of abnormally and persistently elevated, expansive, or irritable
mood lasting more than one week (or less if the patient is hospitalized). During this
time, the patient must exhibit three or more of the following (four or more if the mood is only
irritable): 1) inflated self-esteem or grandiosity, 2) decreased need for sleep, 3) talkativeness
or pressured speech, 4) racing or flighty thoughts, 5) distractibility, 6) increased goal-directed
activity, 7) psychomotor agitation, or 8) excessive involvement in pleasurable but risky
activities. Having one manic episode is sufficient for diagnosing bipolar disorder type 1,
which consists of one or more manic episodes usually, but not necessarily, alternating with
depressive episodes. Before confirming the diagnosis of bipolar disorder, alternative
diagnoses must be ruled out, including substance use, side effect of therapy or medication
use, thyroid disorders, Cushing syndrome, diabetes mellitus, and vitamin B12 deficiency.
Infectious causes of symptoms resembling mania include herpes encephalitis, human
immunodeficiency virus encephalitis, and neurosyphilis. Neurosyphilis occurs in people
who have had chronic, untreated syphilis, and symptoms such as personality changes, mood
disturbances, and psychosis can occur. The rapid plasma reagin test is a rapid diagnostic
test for syphilis antibodies in the serum and it can be used to rule out neurosyphilis in the
patient above. It may be falsely positive in patients with lupus.
Question: What congenital malformations are associated with valproate?

Answer: Neural tube defects (e.g., spina bifida).

The side effects of phenytoin include gingival hypertrophy (A), body hair increase, rash,
folic acid depletion, and decreased bone density. Kidney stones (B) and weight loss (D)
are side effects of topiramate.

Patients being started on valproate should be warned that abdominal pain, nausea,
vomiting, and anorexia can be symptoms of pancreatitis and should seek prompt medical
evaluation
Question: What is the only herbal therapy studied in clinical trials to treat rosacea?
Answer: Licorice.

Topical regimens are first-line therapies for mild papulopustular rosacea because there is less risk of
adverse events, drug interactions, and antibiotic resistance. Topical metronidazole is generally well
tolerated with few local skin adverse reactions, and it is the recommended topical therapy for
rosacea. Topical azelaic acid, sulfacetamide products, and topical acne medications are also
commonly used. Oral tetracyclines, most commonly doxycycline, tetracycline, and minocycline have
been used to treat papulopustular rosacea for more than four decades. Doxycycline is the only
drug approved by the FDA to specifically treat papulopustular rosacea. Three to four weeks of
therapy with a tetracycline is required before substantial improvement occurs; typical duration of
therapy ranges from six to 12 weeks.

Sterile
pyuria and ⊝ urine cultures suggest urethritis by Neisseria gonorrhoeae or Chlamydia
trachomatis.

Chlamydia Cervicitis

 Diagnosis is made by nucleic acid amplification testing (NAAT)

 Most commonly caused by Chlamydia trachomatis

 Treatment is azithromycin
 Comments:

 Most commonly reported sexually transmitted disease in the United States

 Empirically treat for concomitant gonorrhea

 The United States Preventive Services Task Force recommends routine


screening for sexually active women < 24 years of age, and in women > 24 years of
age who are at increased risk

Question: What is the value needed on culture for a clean catch specimen in order to
diagnose a urinary tract infection?

Answer: Greater than 100,000 colony forming units (CFU).

Staphylococcus saprophyticus (D) causes 10-20% of urinary tract infections in females 17–27
years of age and is the second most common cause of urinary tract infections, after Escherichia coli.
Sexual activity increases the risk of S. saprophyticus because bacteria are displaced from the
normal flora of the vagina and perineum into the urethra. Most cases occur within 24 hours of sex,
earning this infection the nickname "honeymoon cystitis." A positive culture urine culture confirms the
diagnosis.

Candida albicans (A) is unlikely because there is no accompanying whitish discharge or itching,
and the patient’s symptoms predate the use of antibiotics. Fungal infections often cause a thick
“cottage cheese” like discharge and can be potentiated by the use of antibiotics.

Question: What is the “ugly duckling” sign?

Answer: The presence of a single lesion that does not match the patient's nevus
phenotype.

Seborrheic keratoses present as well-demarcated, round or oval lesions with a dull,


verrucous surface and a typical stuck-on appearance (D). They are generally
asymptomatic, but chronic irritation due to friction trauma may occasionally cause pruritus,
pain, or bleeding.

Introducing solid foods (D) is recommended between four and six months of age when the
infant is able to sit with support and has control over her head and neck.

Intramuscular administration of prophylactic vitamin K (A) is given to newborns shortly


after birth to prevent vitamin K deficiency bleeding and is not necessary to supplement in a
healthy breastfeeding infant.

Vitamin D is an essential nutrient that plays a key role in calcium homeostasis and bone
health. Infants who are exclusively breastfed should receive 400 international units per
day of vitamin D supplements, beginning within a few days after birth. This
recommendation is based on the low vitamin D content of breast milk, the inconsistency and
unpredictability of cutaneous vitamin D synthesis from sun exposure, and the
disproportionately high frequency of rickets among exclusively breastfed infants. An
exclusively breastfed infant consumes an average of 750 mL of breast milk daily which only
provides 10 to 40 international units/day of vitamin D in the absence of sun exposure or
supplement use. Supplementation is recommended until the infant is weaned and drinks
vitamin D-fortified formula, or after 12 months of age, drinks vitamin D-fortified cow's
milk or fortified plant-based milk.

Crigler-Najjar syndrome (B) is caused by an inherited defect in the gene that encodes
uridine diphosphogluconurate glucuronosyltransferase. Erythrocyte enzymatic defects such
as glucose-6-phosphate dehydrogenase deficiency (C) cause pathologic indirect
hyperbilirubinemia. Primary neonatal jaundice (D) resolves within the first one to two weeks
after birth.

Kernicterus is the chronic and permanent sequela of bilirubin-induced neurologic


dysfunction. Suboptimal breastfeeding is associated with an increased risk of jaundice
and kernicterus. Breastfeeding failure jaundice occurs within the first week of life due to
inadequate intake and fluid loss resulting in hypovolemia, causing hyperbilirubinemia and
hypernatremia.
Absence Seizures (petit mal)

 Patient will be a child 5 - 10-years-old

 Complaining of sudden mental status alteration without motor activity, blank stare

 EEG would show symmetric 3-Hz spike and wave activity

 Treatment is ethosuxamide

 Comments: no aura or postictal state

 Question: What is name of the condition presenting as focal weakness in a part


of the body after a seizure, particularly partial seizures?

 Answer: Todd’s Paralysis.

Complex partial seizures (B) usually start in a small area of the temporal lobe or frontal lobe of
the brain. They quickly involve other areas of the brain that affect alertness and awareness. Even
though the person's eyes are open and they may make movements that seem to have a purpose,
in reality, consciousness is altered. The patient tends to experience amnesia for the event and
automatisms (lip smacking, chewing, swallowing, etc.) may be present. Myoclonic seizures
(C) present as brief, sharp muscle jerks with no impairment of consciousness. Tonic seizures
(D) are characterized as brief, sudden, bilateral, and symmetric stiffening of limbs with rapid
recovery. They are usually seen in neurologically abnormal patients and are often seen in
patients with Lennox-Gastuat Syndrome.

Question: What extra-cardiac manifestation is seen with a genetic type of long QT


syndrome?

Answer: Deafness.
Third degree heart block (B) can develop from isolated single-agent overdose or from
combined or iatrogenic coadministration of digoxin, beta-adrenergic, calcium channel blocking
agents and other antiarrhythmics.

All antipsychotic agents can prolong ventricular repolarization, leading to a prolonged QT


interval, which can in turn lead to torsades de pointes and sudden cardiac death. Although all
antipsychotics can affect ECG intervals, the agents with the greatest propensity to prolong QTc
are thioridazine, pimozide, droperidol, and ziprasidone. The incidence of sudden cardiac death
among patients taking antipsychotics is about twice that of the general population. Physicians
should avoid combining antipsychotic medications with other medications that prolong the
corrected QT interval such as classes I and III antidysrhythmics, tricyclic antidepressants, and
certain antibiotics.

Although pyloromyotomy is the definitive management of infantile hypertrophic pyloric


stenosis, the timing of the surgical correction depends on the infant’s clinical status. Well-
hydrated infants with normal electrolytes may have a surgical repair on the day of
diagnosis. Infants with dehydration or electrolyte derangements should have the
electrolyte imbalances corrected prior to surgery.
Pharyngitis

 Patient will be complaining of a sore throat

 PE will show fever, cervical lymphadenopathy, tonsillar exudate, and no cough

 Most commonly caused by viral > bacteria (GAS, S. Pygoenes)

 Centor Score: Estimates probability that pharyngitis is streptococcal

 Treatment is:

 Viral - supportive

 Bacterial - PCN (first-line)

 PCN allergic:

 Cephalexin, cefadroxil (avoid in individuals with immediate type hypersensitivity to


penicillin)

 Clindamycin

 Azithromycin, clarithromycin (resistance of group A strep to these agents is well-known


and varies geographically and temporally)

 Question: What is the potential harmful outcome of untreated group A strep


pharyngitis?

 Answer: Acute rheumatic fever.

Empiric antibiotic treatment (A) is no longer recommended by IDSA based on symptoms alone.
Common signs and symptoms of streptococcal pharyngitis include sore throat, fever, tonsillar
exudates, and cervical adenopathy. Cough, coryza, and diarrhea are more common with viral
pharyngitis. Available diagnostic tests include throat culture and rapid antigen detection testing.
Testing and treatment is based on the Centor scoring system, which estimates the probability
of streptococcal pharyngitis and the need for antibiotics. The 5 criteria for the Centor scoring
system include: the presence of fever, tonsillar exudate, anterior cervical adenopathy and
absence of cough in patients 3-14 years of age. Patients with a score of zero or 1 are at very low
risk for streptococcal pharyngitis and do not require testing (throat culture or rapid antigen
detection testing) or antibiotic therapy. Testing (using rapid antigen detection test or throat
culture) is optional for patients with a score of 2; positive results warrant antibiotic therapy.
Patients with a score of 3 or higher are at high risk of streptococcal pharyngitis and should have
rapid strep testing or culture or bot

Erythromycin and first-generation cephalosporins are options in patients with penicillin allergy.
Lung Abscess

 Patient will be complaining of several weeks of cough, fever, pleuritic chest pain, weight
loss, and night sweats

 CXR will show area of dense consolidation with an air-fluid level inside a thick-
walled cavitary lesion

 Most commonly caused by aspiration pneumonia

 Treatment is ampicillin-sulbactam, carbapenems, clindamycin


 GoutComments: Can be triggered by loop and thiazide diuretics

 Question: What is the primary treatment of gout?

 Answer: Non-steroidal antiinflammatories.

Many patients without gout or who never develop gout have elevated serum uric acid levels.
Additionally, approximately 15% of patients with acute gout have normal serum uric acid levels.

These crystals cause a marked inflammatory reaction in the joint leading to painful swelling of the
affected joint.

Certain risk factors increase the chance of gout development including: hypertension, diabetes,
obesity, dietary excess, alcohol consumption, and proximal loop diuretics.

Presbycusis is the most common type of Sensorineural Hearing Loss caused by the natural
aging of the auditory system
Question: How are acoustic neuromas diagnosed?

Answer: These intracranial, extra-axial tumors arise from the Schwann cell sheath
invading either the vestibular or cochlear nerve and are diagnosed on MRI.
Otosclerosis (C) is an osseous dyscrasia limited to the temporal bone. Slowly, progressive
conductive hearing loss results. It has been associated with measles.

Sensorineural hearing loss involves problems converting mechanical vibrations to electrical


potential in the cochlea or in auditory nerve transmission to the brain.

The Weber and Rinne tests were designed to distinguish conductive from sensorineural hearing
loss by comparing air and bone conduction
Question: What is Reye syndrome?

Answer: A progressive encephalopathy with hepatic dysfunction.

Cerebral neoplasms (A) may cause chronic daily headache because of increased intracranial
pressure or traction on pain-sensitive structures. Headaches caused by cerebral neoplasms
tend to get worse progressively and tend to be associated with occipital head pain; the
neurologic examination is usually abnormal. Idiopathic intracranial hypertension (B)
typically presents in obese women of childbearing age who present with headaches and
are found to have papilledema on fundoscopic examination.

Myelodysplastic
syndromes
Stem-cell disorders involving ineffective
hematopoiesis Ž defects in cell maturation of
all nonlymphoid lineages. Caused by de novo
mutations or environmental exposure (eg,
radiation, benzene, chemotherapy). Risk of
transformation to AML.
Pseudo–Pelger-Huet anomaly—neutrophils
with bilobed nuclei. Typically seen after
chemotherapy. Clinical features
1. They are often asymptomatic in the early stages. Pancytopenia
may be an incidental
finding on a routine blood test.

2. They may present with manifestations of anemia,


thrombocytopenia, or neutropenia.
Question: What is the most common cause of myelodysplastic syndrome?

Answer: Idiopathic.

References:

Iron deficiency anemia (B) most commonly occurs in women of childbearing age and
typically presents with pallor, alopecia, dry skin, and atrophic glossitis.
Vitamin B deficiency (D) presents with macrocytic anemia and a low serum vitamin B
12 12

level. Pancytopenia is not associated with vitamin B deficiency.


12

atigue, weakness, pancytopenia (anemia, thrombocytopenia, neutropenia), and the


presence of ovalomacrocytosis on peripheral blood smear is strongly suggestive of
myelodysplastic syndrome. Additional findings can include infection, fever, weight loss,
and splenomegaly. Myelodysplastic syndrome consists of a group of malignant
hematopoietic stem cell disorders which is characterized by dysplastic and ineffective
blood cell production. It typically occurs in older adults > 60 years of age. Diagnosis is
typically made with a bone marrow aspirate or biopsy.

Thrush, also known as oropharyngeal candidiasis is a complication that can be found in


patients who have used inhaled glucocorticoids. Patients with thrush typically present
with white plaques on the buccal mucosa, palate, tongue, or oropharynx. Patients may
also have a beefy, red tongue and associated soreness. Topical nystatin therapy is
used as the treatment.

Question: What is the difference between pain during growing pain and the pain
associated with bone cancer?
Answer: Growing pains occur primarily at night and resolve during the day where as
bone cancer pain is throughout the day and night.
There is a strong association between diabetes and Dupuytren's disease.

Question: When is combination therapy with a long-acting angiotensin-converting


enzyme inhibitor plus a long-acting dihydropyridine calcium channel blocker
recommended?

Answer: In patients who have an untreated blood pressure more than 20/10 mm Hg
above goal.

The first-line therapy in patients with hypertension who have heart failure is with an
angiotensin-converting enzyme (ACE) inhibitors (A). Additional indications to use an ACE
inhibitor as a first line therapy include patients with a history of an ST elevation myocardial
infarction, diabetes, systolic dysfunction, or proteinuric chronic kidney disease.

Question: What treatment can be offered to patients whose symptoms from trigger
finger fail to improve with conservative interventions?

Answer: A local glucocorticoid injection.

Stenosing flexor tenosynovitis, known as trigger finger, is one of the most common causes
of hand pain in adults and results when the flexor tendon catches as it moves through a
stenotic sheath resulting in an inability to flex or extend the finger. Patients present with
painless snapping, catching, or locking of one or more fingers during flexion of the affected
digits which often progresses to painful episodes in which the patient has difficulty
spontaneously extending the affected digits. The pain is commonly localized over the volar
aspect of the metacarpophalangeal joint and radiates into the palm or the distal finger.

He states that after he flexes his fingers there is a snapping of his finger when he tries to extend his
finger. Over the past month it has become worse and now he has pain from the finger radiating into
his palm.
Question: What is the reversal antidote for warfarin?

Answer: Vitamin K.

References:

. Intermittent pneumatic compression devices (C) can be used for DVT prophylaxis however they
are not first-line in treatment and should only be used when anti-coagulation therapy is
contraindicated such as in an acute GI or head bleed.

Hip fractures are common among the elderly especially in those that have osteoporosis.
Anticoagulation is necessary in hip fractures as these patients are at an increased risk for deep
vein thrombosis. Without prophylaxis, the rate of deep venous thrombosis (DVT) in these patients
is approximately 50%. Effective agents include unfractionated heparin, enoxaparin, fondaparinux,
and warfarin. Optimal pain control, usually with opioid analgesics, is essential to ensure patient
comfort and to facilitate rehabilitation. It is reasonable to begin preoperative anticoagulation with
unfractionated heparin or low molecular weight heparin (LMWH) as soon as possible after the
fracture occurs because of the increased risk of thromboembolism in patients with fracture. The
guidelines recommend extended prophylaxis with fondaparinux, enoxaparin, or warfarin for 28 to
35 days (four weeks) after surgery, particularly for patients at high risk of thromboembolism. High-
risk patients include those with a history of venous thromboembolism, current obesity, delayed
mobilization, advanced age, or malignancy.

Question: What risk is associated with NPH insulin which is not as common with long-
acting insulin?

Answer: The peak effect of NPH may cause hypoglycemia.

Treatment of patients with type 2 diabetes begins with diet, weight reduction, exercise, and
metformin therapy. These first-line interventions can result in normoglycemia when patients
are compliant and successful. If a patient has persistent hyperglycemia on metformin and a
glycated hemoglobin (A1C) > 8.5 percent, it is recommended to add a second oral or injectable
agent, including insulin, or switch to insulin. Initial therapy with insulin is indicated in patients
who present with A1C > 9.5%, fasting plasma glucose > 250 mg/dL, random glucose
consistently > 300 mg/dL, or ketonuria. For patients who are initiating insulin, starting with
basal, rather than prandial, insulin is recommended. This is predominantly due to greater
patient satisfaction and a lower risk of hypoglycemia. Basal insulin alone is often adequate
for glycemic control since postprandial glycemia can be managed by endogenous insulin
secretion.

Cross-sectional studies look at the prevalence of a disease or an exposure at one moment


in time to give a “snapshot” of the proportion of individuals affected. Cross-sectional studies do
not involve interventions or following individuals over time.

Patients with cirrhosis develop esophageal or gastric varices as a result of longstanding portal
hypertension. Gastrointestinal hemorrhage resulting from variceal bleeding is fatal in approximately
20% of episodes. Bacterial infection is a significant contributor to the cause of death in these
patients. Cirrhotic patients have an impaired immune system as well as increased translocation of
bacteria from the gut into the bloodstream. During an episode of upper gastrointestinal bleeding,
additional bacteria probably translocate contributing to mortality and morbidity. A Cochrane review
demonstrated that the administration of a third-generation cephalosporin to patients with
cirrhosis and upper gastrointestinal bleeding decreases mortality with a number needed to
treat (NNT) of 22. In the acute resuscitation of patients with variceal bleeding, key emergency
medicine principles still apply. These patients often vomit large volumes of blood and require a
secure airway. Adequate large bore intravenous access is required with the availability and
administration of blood products or a massive transfusion protocol. Patients with cirrhosis often have
coagulopathy and thrombocytopenia which needs correction.IMPROVES MORTALITY
Question: What is the mechanism by with gastrografin provides therapeutic relief in
patients with partial small bowel obstruction?
Answer: Gastrografin draws fluid into the lumen of the bowel due to its hypertonicity,
decreasing intestinal wall edema and stimulating intestinal peristalsis.

Postoperative adhesions, malignancies, and hernias are the most common causes of small
bowel obstruction. Management includes volume resuscitation, correction of metabolic
abnormalities, and assessing the of the need for surgery such as ischemia, necrosis,
perforation. Nasogastric suction and intravenous fluids can be used in the treatment of
partial small bowel obstruction and requires frequent reassessments to investigate for
complications.

A diagnosis of tricyclic antidepressant overdose is made based on a history of ingestion,


symptoms and signs consistent with the diagnosis, and characteristic ECG findings.
Laboratory testing does not return quickly enough and, therefore, plays no role in acute patient
management. Neurologic signs of tricyclic antidepressant poisoning include sedation, confusion,
delirium, or hallucinations and demonstration of anticholinergic toxicity including
hyperthermia, flushing, dilated pupils, intestinal ileus, urinary retention, and sinus tachycardia.
Sodium bicarbonate therapy is indicated in patients with tricyclic antidepressant poisoning
who develop widening of the QRS interval > 100 msec or a ventricular dysrhythmia.

Rheumatic Fever

 Patient with a history of GAS infection

 Complaining of fever, red skin lesions on the trunk and proximal extremities, and small,
non-tender lumps located over the joints

 PE will show JONES criteria: Joints, Oh,


no carditis!, Nodules, Erythema marginatum, Sydenham’s chorea

 Labs will show anti-streptolysin O, anti-DNase B, positive throat culture, or positive rapid
antigen test

 Treatment is antibiotics, NSAIDs

 Comments: Modified Jones Criteria for a first episode of acute rheumatic fever: need 2
major or 1 major and 2 minor
A diastolic low-pitched decrescendo murmur best heard at the cardiac apex would be the most
likely auscultatory finding on exam. This woman likely has mitral stenosis secondary to rheumatic
heart disease. Mitral stenosis encountered in women of childbearing age is nearly always rheumatic
in origin. Maternal and perinatal complications during pregnancy in women with mitral stenosis
reflect the unfavorable interaction between the normal cardiovascular changes of pregnancy and the
stenotic mitral valve. Pregnancy is a high flow state and blood volume, cardiac output, and heart rate
are increased. In pregnant patients with mitral stenosis, this increases the pressure across the mitral
valve and can lead to pulmonary edema and other signs and symptoms of heart failure. Although
mitral stenosis is relatively uncommon in developed countries, it is a common condition in pregnant
women with heart disease in areas where rheumatic heart disease is prevalent. Although rheumatic
heart disease may affect other valves, including aortic and tricuspid, the mitral valve is the
predominant lesion and repeated attacks of rheumatic fever over time causes mitral valve stenosis.

Rocky Mountain spotted fever (RMSF)

 Patient with a history of recently being in the woods hiking or camping

 Complaining of abrupt onset of severe headache, photophobia, vomiting, diarrhea,


and myalgia

 PE will show maculopapular eruption on the palms and soles

 Diagnosis is made by skin biopsy

 Most commonly caused by Rickettsia rickettsia

 Treatment is ALWAYS doxycycline, even in children


Chloramphenicol (B) is an alternative treatment, and is used in pregnancy for to treat Rocky
Mountain spotted fever.

he rash appears two to five days after the onset of fever in the majority of cases. The rash
begins as small, blanching, nonpruritic macules that develop into a maculopapular rash and
progress after several days to petechial lesions. It begins around the wrists and
ankles and spreads to the palms and soles, eventually covering the extremities and trunk.

Common laboratory manifestations include thrombocytopenia and hyponatremia

The classic opioid toxidrome involves central nervous system depression, respiratory
depression and miosis. In the central nervous system, opiate use leads to decreased respiratory
effort which leads to hypoxia and hypercapnia. Profound hypoxia may lead to excitatory neurologic
effects like seizures. One particular opiate, meperidine, may also lead to seizures in large quantities
due to its metabolite normeperidine. Opiates stimulate the third nerve nucleus leading to miosis of
the pupils. However, certain opiates do not cause miosis including meperidine, propoxyphene and
diphenoxylate-atropine. The treatment of opiate intoxication is naloxone, a pure opioid competitive
antagonist. Naloxone can be administered intravenously, intramuscularly or intranasally.

Question: An overdose of which antihypertensive mimics opiate intoxication?


Answer: Clonidine.

Naltrexone (D) is an opioid antagonist that is primarily used for patients with alcohol dependence.
Its mechanism of action is not fully understood. It can be used in patients with opioid addiction as
well to help block receptors and prevent additional recreational use. It is not used acutely in the
reversal of opiate intoxication.

Opioid Toxicity

 PE will show respiratory depression, sedation, miosis, bradycardia, hypotension,


hypothermia

 Labs will show prolonged QTc interval

 Treatment is ABC, naloxone

References:

She should receive 50 µg Rh immunoglobulin within 72 hours

Rh immunization is the process by which an Rh-negative woman is exposed to Rh-positive fetal


blood. When this occurs, the pregnant woman develops antibodies against the Rh-positive
antigen. This becomes a problem in future pregnancies if a woman carries another Rh-positive
fetus. If the pregnant woman is Rh-negative, the first step is identification of the father’s blood type. If
the father is known to be Rh-negative, there is no chance of Rh immunization. If the father’s blood
type is Rh-positive, then the developing fetus may also have Rh-positive blood and the
administration of Rh immunoglobulin is recommended. Women with Rh-positive or unknown partners
receive prophylactic Rh immunoglobulin at 28 weeks gestation because during the third trimester of
pregnancy there is a risk of spontaneous maternal-fetal mixing of blood. However, doses are
administered earlier in certain cases including a spontaneous miscarriage as described above. Other
situations include uterine manipulation, threatened miscarriage (much lower risk), surgery for ectopic
pregnancy, amniocentesis and abdominal trauma.

Varicella Zoster (Shingles)

 Patient will be older

 Complaining of a painful, papulovesicular rash preceded by tingling or hyperesthesia

 PE will show a rash with unilateral/dermatomal distribution that does not cross midline

 Tzanck smear will reveal the presence of multinucleated giant cells

 Most commonly caused by reactivation of latent varicella-zoster virus

 Comments: Postherpetic neuralgia: persistent pain > 3 months


. However, treatments for moderate to severe post-herpetic neuralgia include gabapentin and
tricyclic antidepressants. Amitriptyline works by inhibiting reuptake of serotonin and norepinephrine
by presynaptic neuronal membrane, which may increase synaptic concentration in the CNS. The
exact mechanism of gabapentin in the treatment of postherpetic neuralgia is unknown.

Question: In which patients with type 2 diabetes is insulin considered a first-line therapy?

Answer: Patients presenting with hemoglobin A1C > 10%, fasting plasma glucose > 250 mg/dL, random
glucose consistently > 300 mg/dL, or ketonuria.

References:

One complication of metformin use is lactic acidosis. Metformin is contraindicated in

patients with risk factors for developing lactic acidosis such as decreased
renal function, liver disease, alcohol abuse, heart failure, or infection. Patients
a glomerular filtration rate between 30 and 60
with

mL/min or heart failure should discontinue metformin if they develop hypoxemia,


dehydration, or sepsis until the condition has resolved

Question: What physiologic changes are responsible for “hot flashes”?

Answer: Thermoregulatory dysfunction at the level of the hypothalamus induced by


estrogen withdrawal.

Menopausal hormone therapy is recommended for women with moderate to very severe hot
flashes and no contraindications. Women with an intact uterus are treated with both estrogen
and a progestin (A), while those who have undergone hysterectomy can receive estrogen
only (B). To prevent endometrial hyperplasia, all women with an intact uterus need a progestin
in addition to estrogen.

Menopausal hormone therapy is the use of estrogen alone or combined with a progestin to
relieve menopausal symptoms including hot flashes, mood lability/depression, vaginal atrophy,
sleep disturbances related to hot flashes, and joint pains. Menopausal hormone therapy is the
treatment of choice in healthy, symptomatic women who are within 10 years of menopause or
younger than age 60 years and who do not have contraindications including a history of
breast cancer, coronary heart disease, a previous venous thromboembolic event or stroke, or
active liver disease. Women with moderate to severe hot flashes who are not candidates for
hormone therapy can use nonhormonal agents such as selective serotonin reuptake
inhibitors and serotonin-norepinephrine reuptake inhibitors for symptom management.

Urge Incontinence

 Patient will be a woman


 Complaining of the sudden urgent need to urinate, not being able to make it to the
bathroom in time, and nocturia

 Most commonly caused by overactive bladder (detrusor overactivity)

 Treatment is anticholinergics (oxybutynin)

 Question: Which medication classes are used to treat urge incontinence?

 Answer: Anticholinergic drugs, antispasmodics, and tricyclic antidepressants

Overflow incontinence (B) is caused by impaired detrusor contractility or bladder outlet obstruction
resulting in overdistension of the bladder. Chronic overflow incontinence is common in men because
of prostatic hyperplasia, but it is uncommon in women.

rge incontinence is the most common type of incontinence seen in both elderly women and
men

ontrast-enhanced esophagography (B) is indicated if emesis or retching is the precipitating event


and the esophagus is thought to be ruptured. This is seen in disorders such as Boerhaave's
syndrome. V/Q scan (D) and computed tomography angiography (A) are indicated in the
suspicion and workup for a pulmonary embolism. The symptoms of pulmonary embolism and
pneumothorax are similar however the classic finding of absent breath sounds is not seen in
pulmonary embolism. In addition, a patient presenting with pulmonary embolism should have other
risk factors mentioned such as recent travel, oral contraceptive use in females, and leg edema for
example.

Respiratory distress and decreased or absent lung sounds on the ipsilateral side are often seen
on physical exam and should prompt the diagnostic workup for pneumothroax. History and physical
findings are enough to make the diagnosis however radiography is often used to confirm the
diagnosis. Chest radiography is the first investigation performed to assess pneumothorax, because
it is simple, inexpensive, rapid, and noninvasive; however, it is much less sensitive than chest
computed tomography (CT) scanning in detecting blebs or bullae or a small pneumothorax.

Cervical cancer—Quadrivalent HPV vaccine (Gardasil) protects


against high-risk HPV,
approved in females ages 9 to 26 years old. Begin annual pap
smears at age 21 even if
not sexually active.

2. Cervical cancer
a. Start at age 21, irrespective of sexual history
b. Age 21 to 29, Pap smear every 3 years
c. Age 30 to 65, Pap smear every 3 years or Pap smear
HPV testing every
5 years
d. Can discontinue screening at age 65 if adequate
negative prior screening
(3 negative Pap smears or 2 negative Pap smears with
negative HPV testing

Begin annual pap smears at age 21 even if


not sexually active.
Cervical cancer screening is critical in women infected with human immunodeficiency virus
(HIV) since the risk of cervical intraepithelial neoplasia is higher in HIV-infected women.
Because both HIV and HPV are sexually transmitted, HIV-infected women are more likely to
have persistent HPV infection, and persistent infection with oncogenic human papillomavirus
subtypes is a major factor in the pathogenesis of cervical disease. Women with HIV, or who are
otherwise immunocompromised, should undergo cervical cancer screening twice in the first
yearafter diagnosis of HIV infection and then annually if the test results are normal. A screening
colposcopy is recommended at the initial evaluation. The need for subsequent examinations
depends on cervical cytology results.

Question: What are the 3 values used to diagnose diabetes mellitus type 2?

Answer: Need 2: Fasting blood sugar > 126 mg/dL, HBA1c of 6.5%, glucose tolerance
test results > 200 mg/dL or a random plasma glucose level > 200mg/dL + symptoms of
hyperglycemia.

According to the recommendations the hepatitis B vaccine should be administered to


all previously unvaccinated adults aged 19-59 with diabetes mellitus, as soon as
possible after the diagnosis of diabetes is made. Hepatitis B vaccine should also be
given to patients whom are at high risk for sexually transmitted diseases, health care
personnel, chronic liver disease and HIV infected patients. Three doses should be
administered at least 6 months apart.

Hepatitis A vaccine (A) is recommended for any person with chronic liver disease or
traveling and working in endemic areas. This vaccine is not indicated for a patient with
diabetes. Meningococcal vaccine (C) is given to adults with asplenia, freshman
college students in dormitories or military personnel. Varicella zoster vaccine (D) is
recommended for adults 50 years of age or older regardless of whether they report a
prior episode of herpes zoster.

The CDC recommends hepatitis B vaccination for all unvaccinated adults


withdiabetes younger than 60 years of age. ... If you think you have already beenvaccinated,
confirm with your doctor
Topical regimens are first-line therapies for mild papulopustular rosacea because there is less risk of
adverse events, drug interactions, and antibiotic resistance. Topical metronidazole is generally well
tolerated with few local skin adverse reactions, and it is the recommended topical therapy for
rosacea. Topical azelaic acid, sulfacetamide products, and topical acne medications are also
commonly used.

Doxycycline is the only drug approved by the FDA to specifically treat papulopustular
rosacea.

Hypertriglyceridemia

 Xanthomas

 Tendinous xanthomas

 Corneal arcus

 Treatment:

 150-199: lifestyle modifications

 200-499: consider pharmacologic therapy for high-risk pts

 >500: pharmacologic therapy

Eruptive xanthomas typically appear suddenly as multiple erythematous-yellow, dome-shaped


papules on the extensor surfaces of the extremities, buttocks, and hands. The lesions are small
and may have an inflamed base, or be tender or pruritic. The condition may be the first presenting
sign of an underlying lipid disorder and occurs almost exclusively in the setting
of hypertriglyceridemia, with triglyceride levels usually in the thousands

A one-time screening for abdominal aortic aneurysm (AAA) with ultrasonography (A) is
recommended in men aged 65 to 75 years who have a smoking history

Pneumococcal vaccine is recommended for all adults aged 65 or older. Patients aged 2 years
or older who are immunocompromised or who have chronic diseases should also receive the
pneumococcal vaccine. Adults aged 65 or older should be revaccinated if they received their first
pneumococcal vaccine greater than five years ago or if they are considered high risk. The
pneumococcal vaccine is also a recommended routine vaccine series given in childhood starting at 2
months of age. Patients undergoing a splenectomy should also receive this vaccine. This patient has
COPD, an underlying chronic disease, and therefore should receive the pneumococcal vaccine
today and should be revaccinated at age 65.

Atopic dermatitis (eczema) is a chronic, pruritic, inflammatory skin disease. Clinical


features of atopic dermatitis include skin dryness, erythema, oozing and crusting, and
lichenification. Pruritus is a common symptom and is responsible for much of the disease
burden. The management of atopic dermatitis requires elimination of exacerbating factors,
restoration of the skin barrier function and hydration of the skin, and pharmacologic treatment of
skin inflammation. For patients with mild atopic dermatitis, low-potency topical corticosteroids
such as desonide or hydrocortisone with the application of emollients are recommended. For
patients with moderate disease, medium- to high-potency corticosteroids such as
fluocinolone, triamcinolone, or betamethasone are recommended.

Question: What is the first line intravenous antibiotic therapy for patients with
endometritis?

Answer: Clindamycin and gentamicin.

The patient is suffering from post-partum endometritis for which history of Cesarean section is
the most important risk factor. Endometritis classically occurs 2-3 days post-partum and is
characterized by fever, foul-smelling lochia, abdominal and pelvic pain, abnormal vaginal
bleeding, uterine tenderness and leukocytosis

Question: When can anticoagulation be discontinued in patients with a provoked


pulmonary embolism secondary to a transient risk factor?

Answer: 3 months.

For most patients with suspected pulmonary embolism, it is important to determine the pretest

probability. Using the Wells Criteria , obtaining a D-dimer level would be the next
fBased on the Wells Criteria, this patient has tachycardia with pulse >100 (1.5 points) and a
history of surgery in the previous four weeks (1.5 points). Additional findings included in the
Wells Criteria include physical findings suggestive of deep venous thrombosis (3 points), no
alternative diagnosis to better explain the illness (3 points), immobilization ≥ 3 days (1.5 points),
prior history of deep venous thrombosis or pulmonary embolism (1.5 points), presence of
hemoptysis (1 point), and presence of malignancy (1 point). The history of travel can increase
the suspicion of pulmonary embolism, but does not specifically add to the Wells Criteria to
determine pretest probability.

CT pulmonary angiography (B) is indicated as the next step if the patient had a high pretest
probability for pulmonary embolism, but not for patients with a low or intermediate pretest
probability. Ventilation perfusion scan (D) is indicated only if CT pulmonary angiography is not
feasible or is inconclusive regardless of pretest probability.

Question: Which type of heparin-induced thrombocytopenia (HIT) is not associated


with thrombosis and patients can be managed expectantly without discontinuation of
heparin?

Answer: HIT type I.


The 4 Ts score is used to estimate the likelihood of heparin-induced thrombocytopenia based
on readily available clinical data, including the degree of thrombocytopenia, timing of
platelet count drop, presence of thrombosis, and absence of other causes of
thrombocytopenia.

Question: What is the most common mechanism for the development of septic arthritis
in children?

Answer: Hematogenous spread.

he knee is the most commonly affected joint in children and the hip is the second most common. It
generally presents with acute onset of fever, irritability, pain, and refusal to bear weight or move the
affected joint. Generally the area is warm, erythematous, and swollen. In cases of hip involvement, it
is often held in flexion, abduction and external rotation. Younger children will have more nonspecific
symptoms. Generally the WBC count, ESR and CRP will be elevated. Arthrocentesis and synovial
fluid analysis should be performed. In cases of septic arthritis, the synovial fluid is turbid or purulent,
WBC count > 50,000 cells/mL with a PMN predominance, low glucose, elevated protein and lactate.
The patient should be immediately started on intravenous antibiotics, evaluated by orthopedics for
surgical intervention and admitted to the hospital.

The diagnosis is aided by the use of a Wood’s light, in which the depigmented areas emit a
bright blue-white fluorescence and are sharply demarcated. A skin biopsy is not routinely
required for the diagnosis of vitiligo. On histology, vitiligo reveals complete loss of melanin
pigment in the epidermis and absence of melanocytes, and lymphocytes at the border of the
lesion.

Topical glucocorticoids and photochemotherapy are used to promote


repigmentation
with varying degrees of success

Neuroleptic agents such as haloperidol are generally used to treat severe agitation in
patients with delirium.
This patient has compartment syndrome of his left anterior leg. Compartment syndrome occurs
when the tissue pressure within a closed muscle compartment exceeds the perfusion pressure and
results in muscle and nerve ischemia. Compartment syndrome may affect any compartment,
including the hand, forearm, upper arm, abdomen, buttock, and entire lower extremity. Almost any
injury can cause this syndrome, including injury resulting from vigorous exercise. Patients with
compartment syndrome typically present with pain whose severity appears out of proportion to
the injury. The pain is often described as burning. The pain is also deep and aching in nature and is
worsened by passive stretching of the involved muscles. The patient may describe a tense feeling in
the extremity. The traditional 5 P's of acute ischemia in a limb (pain, paresthesia, pallor,
pulselessness, poikilothermia) are not clinically reliable; they may manifest only in the late stages of
compartment syndrome, by which time extensive and irreversible soft tissue damage may have
taken place. Compartment syndrome has been found in soldiers and athletes without any trauma.
This can be acute or chronic, with acute compartment pressures as high as those found in severe
trauma. Chronic compartment syndrome (CCS) is a recurrent syndrome during exercise or work.
CCS is characterized by pain and disability that subside when the precipitating activity is stopped but
that return when the activity is resumed. If compartment syndrome is suspected, check
intracompartmental pressure, even in the absence of any trauma history. The definitive therapy for
compartment syndrome is emergent fasciotomy (compartment release), with subsequent fracture
reduction or stabilization and vascular repair, if needed. The goal of decompression is restoration of
muscle perfusion within 6 hours.
Question: What is the most common cause of chronic obstructive pulmonary disease
exacerbations?

Answer: Respiratory infections.

The evaluation of hemoptysis should an include an assessment of oxygenation and a thorough


history of the events leading up to the episode. Patients should be questioned as to whether they
have had any association with symptoms of acute bronchitis, acute exacerbation of chronic
bronchitis, or bronchiectasis, such as a change in sputum or blood streaking superimposed upon
purulent sputum. Additionally, the duration and amount of hemoptysis should be quantified.
Following the history and physical examination, all patients presenting with hemoptysis should
receive a chest radiograph which can identify a variety of causes of hemoptysis.

Question: What are the five symptom types diagnostic of schizophrenia?

Answer: Delusions, hallucinations, disorganized speech, disorganized or catatonic


behavior, and negative symptoms such as avolition.

he first-generation typical antipsychotics are mostly associated with low risk of


metabolic changes, but they are not often a preferred choice because they carry an
increased risk of extrapyramidal symptoms such as tardive dyskinesia and akathisia.
Among the atypical antipsychotics, the choice of aripiprazole is most appropriate for this patient
because aripiprazole has one of the lowest risks of metabolic effects of all the atypical
antipsychotics. Particularly, aripiprazole is the atypical antipsychotic least associated with
increased dyslipidemia

Congenital rubella findings include “blueberry


muffin” appearance due to dermal extramedullary
hematopoiesis.

Erysipelas (B) is a superficial infection of the skin that only affects the dermis and is characterized
by sharply demarcated borders.

Toxic epidermal necrolysis (D) is a severe dermatologic disorder that is usually drug induced. It is
characterized by widespread erythema, necrosis, and bullous detachment of the epidermis and
mucous membranes. By definition, it must involve at least 30% of the body surface.

Cellulitis is more localized and does not affect layers deeper than the subcutaneous tissues

Endocarditis should be suspected in any patient with unexplained fevers, night sweats, or signs of
systemic illness, particularly if any of the following risk factors are present: a prosthetic heart valve,
structural or congenital heart disease, intravenous drug use, and a recent history of invasive
procedures. Clinical history consistent with infectious endocarditis includes the combination of a prior
cardiac lesion and evidence of a recent source of bacteremia. In general, right-sided
infectious endocarditis is far less common than left-sided, and most cases occur on the tricuspid
valve in persons who use injection drugs. The American College of Cardiology and the American
Heart Association recommend that echocardiography be performed to identify valvular abnormalities
in all patients in whom there is moderate or high suspicion of endocarditis. Staphylococcus aureus is
the most common cause of acute infectious endocarditis worldwide. Additionally, the most common
cause of tricuspid valve endocarditis is intravenous drug abuse, and Staphylococcus aureus is the
infecting organism in 80% of tricuspid valve infections.

Benign paroxysmal positional vertigo (BPPV) is the most common form of positional vertigo,
accounting for nearly one-half of patients with peripheral vestibular dysfunction. BPPV is
most commonly attributed to canalithiasis within a semicircular canal. Observing nystagmus
during a provoking maneuver solidifies the diagnosis of BPPV in patients with a typical history.
Treatment involves particle repositioning maneuvers. In patients with posterior canal BPPV, the
best evidence of efficacy is with the Epley maneuver.

Question: What are the current recommendations regarding clinical breast


examinations as part of screening for average-risk women?

Answer: Clinical breast examinations as a screening tool are not recommended unless
a woman presents with breast complaints or abnormalities.

Beginning routine screening at age 45 (A) would be incorrect since breast cancer screening
with mammography for average-risk women begins at age 50. Screening is typically
recommended every two years unless the prior screening identifies a concerning finding.
Suggesting mammography now (C) would be inappropriate since screening is not
recommended in women younger than age 40 as the incidence of breast cancer is low, and the
performance characteristics of mammography are poor. Mammography is the primary modality
for breast cancer screening in average-risk women. Ultrasound (D) and magnetic resonance
imaging are reserved for further evaluation of findings on mammography.

The majority of breast cancers in the United States are diagnosed as a result of an abnormal
mammography screening studies. The recommendations regarding when a woman should
begin screening depends on risk factors included in the patient’s history including a personal
history of ovarian, peritoneal, or breast cancer, a family history of breast, ovarian, or peritoneal
cancer, genetic predisposition, and radiotherapy to the chest between ages 10–30. Most women
with breast cancer in a first-degree relative but without a known genetic syndrome are
categorized as moderate-risk. Women with moderate risk are screened with the same
approach as women with average-risk, which is with mammography beginning at age 50 as
recommended by the U.S. Preventive Services Task Force. However, it is appropriate to
make an individualized decision based on patient preferences and values for women age
40 to 49 who may opt for screening if they present with substantial concerns about breast
cancer risk and accept the possibility of a false-positive result or overdiagnosis and the resulting
evaluation and treatment.

Electroconvulsive therapy (C) is an effective treatment for severe, unremitting depression. It is


particularly effective in the elderly and has been shown to improve cognitive functioning. This
therapy is not applicable to the patient above, as she does not fit the criteria for major
depressive disorder. Sertraline (D) is a selective serotonin uptake inhibitor used for first-line
treatment of major depressive disorder. There is no evidence that antidepressants have any
effect on mood changes associated with adjustment disorder.

The main treatment for adjustment disorder is psychotherapy, particularly short-term


therapies such as brief psychodynamic psychotherapy or short-term cognitive behavioral
therapy. Importantly, adjustment disorder has a good prognosis, especially in patients who
have no history of mental illness. Many patients recover with time due to removal of the stressor
or due to their own resilience and adaptive skills.
Pterygia are fibrovascular proliferations thought to be triggered by ultraviolet light exposure. They
occur more commonly in young males from hot, dry climates. Pterygia are characterized by their
shape which is widest at the bulbar conjunctiva and narrowest at the cornea. Visual impairment may
occur in large pterygia, but most are asymptomatic. Surgical removal is necessary once vision is
impaired.

Idiopathic is the most common classification of dilated cardiomyopathy and is considered


when all detectable causes have been excluded (except genetic causes)

Question: What class of medications has been recommended to slow progression in


patients with left ventricular enlargement and a reduced left ventricular ejection fraction?

Answer: Angiotensin-converting enzyme inhibitors.


This patient has a deep vein thrombosis (DVT) in her right leg and also has two hypercoagulable
states which increase her risk for blood clots. Symptoms of deep vein thrombosis include edema, leg
pain, tenderness and warmth. Wells criteria should initially be used to determine this patients risk
and either a D-dimer or imaging studying should be performed depending on the risk stratification.
A venous compressive ultrasound is the diagnostic imaging study of choice which should be
performed in this patient due to the fact that she has two hypercoagulable disorders which puts her
at risk for blood clots. Laboratory analysis has been used in aiding the diagnosis of venous
thrombosis. Protein S, protein C, antithrombin III (ATIII), factor V Leiden, prothrombin 20210A
mutation, antiphospholipid antibodies, and homocysteine levels can be measured.
For patients with a first-episode venous thrombosis and documented antiphospholipid antibodies or
2 or more thrombophilic conditions at least 12 months of anticoagulation treatment is indicated.

Question: What is due to the compression of the posterior tibial nerve and causes
neuropathic pain and numbness in the posteromedial ankle and heel?

Answer: Tarsal tunnel syndrome.


Calcaneal apophysitis, also known as Sever disease, is the most common etiology of heel pain
in children, usually occurring between five and eleven years of age. It is thought that in these
children the bones grow faster than the muscles and tendons

Treatment involves decreasing pain-inducing activities, anti-inflammatory or analgesic medication if


needed, ice, stretching and strengthening of the gastrocnemius-soleus complex (calf muscles), and
the use of orthotic devices.

A clinically significant fever in children younger than 36 months is a rectal temperature of at least
100.4°F (38°C). Guidelines for the management of fever in children have been based on age groups:
neonates (younger than 28 days); young infants (up to two months or three months); and older
infants and young children (up to 36 months). Any child younger than 28 days old with a
fever and any child who appears toxic, regardless of age, should undergo a complete sepsis workup
and be admitted to the hospital for observation until culture results are known or the source of the
fever is found and treated. The standard septic workup for febrile infants younger than 29 days
should include complete blood count with differential, lumbar puncture, blood culture, chest
radiography, urinalysis, and urine culture
Ischemic colitis is a syndrome caused by inadequate blood flow through the mesenteric
vessels, resulting in ischemia and possible gangrene of the bowel wall. Patients typically present
with fairly acute onset crampy abdominal pain with tenderness over the affected bowel. Patients
may have bloody diarrhea or passage of frank blood although it is not usually enough to warrant
transfusion. The presentation with ischemic colitis differs from acute mesenteric ischemia which
presents as pain that is disproportionate to physical examination findings. Risk factors include
a history of atherosclerotic disease at other sites, such as coronary artery disease or
cerebrovascular disease, advanced age, sepsis and extreme exercise.

Most cases resolve with supportive care (eg. IV fluids and bowel rest).
Angiodysplasia (A) is a degenerative lesion of previously healthy blood vessels found most
commonly in the cecum and results in hematochezia. Bleeding is usually chronic or recurrent and, in
most cases, low grade and painless because of the venous source. Diverticular bleeding (B) is the
most common cause of lower gastrointestinal hemorrhage. Patients with diverticular bleeding usually
present with an abrupt onset of painless rectal hemorrhage. Occasionally, patients may present with
mild abdominal cramping or the urge to defecate, secondary to blood within the colon

Social anxiety disorder—exaggerated fear of embarrassment in social situations (eg, public


speaking, using public restrooms). Treatment: CBT, SSRIs, venlafaxine.

For only occasional


anxiety-inducing situations, benzodiazepine or β-blocker.

Performance-only social anxiety disorder, on the other hand, refers to intense fear
surrounding performance situations in which the patient may be scrutinized by others.
This subgroup is characterized differently than classic social anxiety disorder, including having a
lower rate of heritability, a later age of onset, less overall social impairment outside of the
anxiety-inducing events, and a positive response to treatment with β-adrenergic receptor
blocker

Borderline Unstable mood and interpersonal relationships,


impulsivity, self-mutilation, suicidality, sense
of emptiness; females > males; splitting is a
major defense mechanism.
Treatment: dialectical behavior therapy.
The X-ray shows a right-sided posterior hip dislocation. Dislocated hips are frequently seen in elderly
patients and are considered medical emergencies. They are also more common in patients who
have had a hip replacement. Dislocation occurs when the ball–shaped head of the femur comes out
of the cup–shaped acetabulum set in the pelvis. Posterior (vs anterior) hip dislocations are the
most common. On physical examination the limb is shortened, and the hip flexed, the foot is in
internal rotation.

A 3-year-old boy presents in severe respiratory distress. His mother informs you that he has
been ill for the last 5 days, initially with a low-grade fever and “barky cough.” He was seen at
an urgent care facility 4 days ago and given a “breathing treatment” and discharged on
steroids. He has become progressively worse despite compliance with the steroid regimen,
which prompted his mother to call an ambulance this morning. He is otherwise healthy and
up-to-date on his immunizations. On examination, the child is toxic in appearance and febrile.
His oropharynx is clear. You hear both inspiratory and expiratory stridor. What is the most
likely diagnosis?
he patient is suffering from acute bacterial tracheitis. Bacterial tracheitis is the result of severe
inflammation of the epithelial lining of the trachea leading to thick mucopurulent secretion production.
This clinically manifests as viral prodrome with fever, URI symptoms, barky cough and stridor that
intensifies and progresses to include a toxic appearing child with signs of airway obstruction,
inspiratory and expiratory stridor, cyanosis, and severe respiratory distress. Another clue is that the
child has been treated with medications (aerosolized epinephrine and steroids) for croup and has not
improved clinically. Bacterial tracheitis is most common in children between the ages of 3 to 5 years.
Most patients require orotracheal intubation for respiratory distress and ICU admission. The patient
should be started on broad-spectrum intravenous antibiotics.

Question: What intervention is both diagnostic and therapeutic in the management of


bacterial tracheitis?

Answer: Bronchoscopy.

Bacterial superinfection caused by Staphylococcus aureas or Strep spp.


Caused by long-standing, often untreated, primary viral upper respiratory infection (croup)
caused by parainfluenza virus
Epidemiology
occurs in ages 3 months to 2 years old
often a complication of croup
Presentation
Symptoms
Croup (B) is the most common cause of upper airway distress and obstruction in children between 6
months to 6 years of age with peak incidence at 2 years of age. Croup begins as a prodrome of low-
grade fever and URI symptoms and is characterized by a barky cough, inspiratory stridor, and
hoarse voice. Children are less toxic in appearance and rarely develop respiratory failure. The
mainstays of treatment are steroids and aerosolized epinephrine. Epiglottitis (C) is characterized by
abrupt onset of fever and sore throat and children classically present with difficulty in breathing,
anxiety, stridor and drooling. This is less common in vaccinated children, such as the patient above
and typically occurs in slightly older children. There is generally not a prodrome associated with
epiglottitis. Peritonsillar abscess (D)occurs more commonly during adolescence and presents with
trismus, unilateral sore throat, fever, tonsillar asymmetry, and uvula deviation away from the affected
tonsil. The age of this patient and normal oropharynx examination make this diagnosis very unlikely.

24 milliliters of 25% dextrose

A complication of atrial fibrillation is the development and subsequent embolization of atrial


thrombi. Therefore, oral anticoagulation is recommended for patient with atrial fibrillation. The
benefit from anticoagulation exceeds the risks for almost all patients with a CHA2DS2-
VASc score ≥ 2. A direct thrombin inhibitors or factor Xa inhibitors are preferred rather
than warfarin in patients with nonvalvular atrial fibrillation. However, warfarin is
recommended in patients with chronic severe kidney disease, with an estimated glomerular
filtration rate less than 30 mL/min/1.73m 2

Question: What lab value needs to be closely monitored in a patient started on


warfarin?

Answer: International normalized ratio (INR).


Endometriosis is characterized by endometrial glands and stroma found outside of the uterine
cavity. While endometriosis itself is a nonmalignant process, the abnormal tissue may cause
dysmenorrhea, dyspareunia, chronic pain, and infertility in some women. Physical
examination findings vary based on the location and size of the ectopic tissue. The definitive
diagnosis is made by histologic evaluation of a biopsied lesion. The first-line treatment for
women with endometriosis-related pain are nonsteroidal anti-inflammatory drugs and
combined (estrogen and progestin) contraceptives

Question: What treatment is recommended for patients with severe symptoms from
endometriosis?

Answer: Gonadotropin-releasing hormone (GnRH) agonist with add-back hormonal


therapy.
Pelvic floor physical therapy (C) is a treatment for women who experience pain with
penetration which typically presents with superficial introital pain rather than deep dyspareunia.
Progressive vaginal dilation (D) is used in the treatment of introital pain rather than deep
dyspareunia.

Pelvic floor physical therapy (C) is a treatment for women who experience pain with
penetration which typically presents with superficial introital pain rather than deep dyspareunia.
Progressive vaginal dilation (D) is used in the treatment of introital pain rather than deep
dyspareunia.

Alopecia Areata

 Patient with a history of an autoimmune disorder

 Complaining of hair loss

 PE will show patches of smooth, non-scarring hair loss with patches of smaller
hairs termed “exclamation hairs”

 Most common cause is autoimmune

 Treatment is intralesional corticosteroi

This patient’s examination is consistent with female pattern hair loss. Up to 50% of women will
experience female pattern hair loss during their lifetime. Patients usually present with hair thinning
over the central area of the scalpand widening of the midline part, but with preservation
of the frontal hairline. Women who also have abnormal menses, history of infertility, hirsutism,
unresponsive cystic acne, virilization, or galactorrhea should have a targeted endocrine work-up for
hyperandrogenism. Evaluation for iron deficiency, thyroid disease, and syphilis should be considered
because they can contribute to hair thinning or generalized hair loss. Topical minoxidil 2% is the
only treatment approved by the FDA for treating female pattern hair loss in women over 18 years of
age. Approximately 90% of such women report a modest decrease in hair loss with this treatment. A
hyperandrogenic state may limit the success of treatment with minoxidil, and, in these women,
spironolactone 100 to 200 mg daily may slow the rate of hair loss.
The patient has a branchial cleft cyst. A branchial cleft cyst is a congenital epithelial cyst that
results from the incomplete resorption of the embryonic branchial apparatus during development.
The cysts are located laterally, anterior to the border of the sternocleidomastoid near the angle
of the mandible.

Depression that is resistant to treatment with antidepressant monotherapy is an indication


to attempt treatment with antidepressant combinations. Combinations should include
antidepressants with different mechanisms of action. In this patient who is tolerating
sertraline (a selective serotonin reuptake inhibitor) and has concerns about weight gain and
sexual dysfunction, adding bupropion would be an effective augmentation strategy.

Adding mirtazapine (B) is associated with weight gain and therefore would not be the best
option for this patient. Adding phenelzine (C), a monoamine oxidase inhibitor, is
contraindicated as an augmenting agent in patients taking a selective serotonin reuptake
inhibitor due to risk of serotonin syndrome. Since this patient has tried two selective serotonin
reuptake inhibitors and has improved on sertraline, it is preferable to choose an augmenting
agent from a different class rather than switching to paroxetine

Question: What physical exam findings are present in a patient with serotonin
syndrome?

Answer: Hyperthermia, agitation, ocular clonus, tremor, akathisia, deep tendon


hyperreflexia, inducible or spontaneous clonus, muscle rigidity, dilated pupils, dry
mucous membranes, increased bowel sounds, flushed skin, and diaphoresis.
Coarctation of the Aorta

 PE will show higher blood pressure in the arms than in the legs

 EKG will show LVH

 CXR will show notching of ribs

 Diagnosis is made by echo

 Treatment is balloon angioplasty with stent placement, or surgical correction

 Comments: Associated with Turner's syndrome

Selection of a contraceptive method must take into account the woman's preferences, her
time frame for pregnancy, and contraceptive efficacy, safety, and side effects. The most effective
method aside from sterilization are long-acting reversible contraception (intrauterine
contraception and contraceptive implants). These methods are associated with the lowest
pregnancy rates regardless of the population studied because their effectiveness is minimally
influenced by the patient's actions or adherence. Noncontraceptive benefits of
levonorgestrel-releasing IUDs include reduction in menorrhagia, anemia, dysmenorrhea,
endometriosis-related pain, endometrial hyperplasia, PID, and cervical cancer. Placement

oflevonorgestrel-releasing IUDs for these noncontraceptive indications


is off-label but a common practice that is supported by strong medical evidence.

Copper IUDs (B) may worsen symptoms in women with menorrhagia, dysmenorrhea, or
endometriosis-related pain, whereas levonorgestrel-releasing IUDs reduce menstrual flow and
dysmenorrhea, which would benefit this patient.

Question: What structure is interrupted or occluded during a vasectomy?

Answer: The vas deferens.

The contraceptive transdermal patch (A) and oral contraceptives (D) are associated with a
very low pregnancy rate if they are used consistently and correctly, but actual pregnancy rates
are substantially higher because of inconsistent and incorrect use. In this patient who travels for
work and switches time zones, long-acting reversible contraception would be a more reliable
method.

The acronym RICE (rest, ice, compression, elevation) should be followed for the first two to
three days. Compression with an elastic bandage should be applied early to minimize
swelling. Ankle splints or braces are used to limit joint motion and allow early weight bearing
while protecting against reinjury.
The Ottawa ankle rules were developed to determine which acute ankle
injuries required radiographic images. Obtaining a foot series (C) is only
indicated for patients who have pain in the midfoot zone and have bone
tenderness at the base of the fifth metatarsal or at the navicular, or are
unable to bear weight both immediately after the injury and for four steps
during evaluation. Obtaining an ankle series (D) is only indicated for
patients who have pain in the malleolar zone and have bone tenderness at
the posterior edge or tip of the lateral or medial malleolus or are unable to
bear weight both immediately after the injury and for four steps during
evaluation. An emergency orthopedic evaluation (B) would be necessary
in patients with neurovascular compromise including findings of decreased
sensation, motion, or circulation.
Imaging
Radiography
indication
depend on the Ottawa ankle rules
in patients with malleolar zone pain and bone tenderness at the lateral or m
in patients who are unable to bear weight either
immediately after the injury
after taking a few steps in the emergency department

Treatment
Conservative
RICE (Rest, Ice, Compression, and Elevation) therapy
indication
initial treatment for an ankle sprain
physical therapy
indication
follows RICE therapy
Operative
ligament repair
indication
considered in certain clinical conditions (e.g., severe sprain of the d
ankle mortise)

Non-accidental Trauma
 Young children more commonly have non-accidental rib fractures
 Posterior rib fractures and multiple fractures in various stages of healing are the fractures
most consistent with abusive injury
 Metaphyseal chip fractures (also known as bucket-handle fractures) are pathognomonic
for abuse

 Prematurity, colic, socioeconomic stress, and prior medical conditions are known risk
factors for abuse

he varicella zoster vaccine is recommended for immunocompetent patients over the age of 50.
There is not sufficient research at this time in immunocompromised hosts and therefore it is
contraindicated in this patient due to her immunocompromised state. This patient is considered
immunocompromised since she is taking infliximab.

"MR VZ MAPSY"
Mumps, Rubella, Varicella
Zoster, Measles, Adenovirus
(not attenuated), Polio
(SABIN), Small pox, yellow
fever
Management of patients with disseminated intravascular coagulation involves establishing
intravenous access peripherally with at least two large intravenous catheters and
infusing crystalloid, blood products, or both to begin fluid resuscitation.

oxytocin and perform fundal massage (B) in the case of postpartum hemorrhage caused by
uterine atony, but would be insufficient treatment for a patient with disseminated intravascular
coagulation.

Disseminated intravascular coagulation may also occur in the setting of postpartum


hemorrhage which is characterized by cumulative uterine blood loss ≥ 1000 mL or uterine
bleeding associated with symptoms of hypovolemia. Patients with disseminated intravascular
coagulation may present with severe uterine bleeding or bleeding from skin or mucosa. Women
with disseminated intravascular coagulation have thrombocytopenia, coagulation factor
consumption, and fibrinolysis. Laboratory findings include prolonged prothrombin time and
activated partial thromboplastin time, low fibrinogen, and increased D-dimer.

All adults should receive the tetanus diphtheria toxoid boster td vaccine every 10 years.
Adults should also receive the tetanus diphtheria acellular pertusis TDAP vaccine as a one
time dose in place of aTd. If a patient has nt received Tdap as an adult or if the prior
vaccine history is unknown the CDC recommends THAT Tdap be given followed by Td
every 10 years after.

The physical examination finding of hepatomegaly suggests underlying congestive heart


failure and therefore a cardiogenic source of neonatal shock.

Fever (A) suggests septic shock from an underlying infection

Scleroderma is also known as progressive systemic sclerosis, a disease much more common
in women than men.

Ocrelizumab is an approved disease modifying treatment of primary progressive multiple


sclerosis. Primary progressive multiple sclerosis is a syndrome that is equally common in men
and women and affects about 10-15% of multiple sclerosis patients. The McDonald Criteria for the
diagnosis of primary progressive multiple sclerosis requires evidence of lesions disseminated in
time in the brain, spinal cord or positive CSF involvement or both. Early recognition and diagnosis is
crucial since ocrelizumab can stop and reverse the nerve damage associated with primary
progressive multiple sclerosis.

In addition, prophylactic antibiotic eye therapy reduces the risk of gonococcal conjunctivitis.
Prophylaxis with erythromycin ophthalmic ointment can be administered up to one hour after
birth.
Question: At what age should the herpes zoster vaccine be administered?

Answer: Patients 50 years and older. it is contraindicated in those who are


immunosuppressed.

Postherpetic neuralgia is the most common complication of herpes zoster. It is characterized


by pain in a dermatomal distribution that is burning and electric in nature, usually sustained for
at least 90 days after the rash.

Oral medications include gabapentin , pregabalin and tricyclic antidepressants are the
mainstay of treatment.

Esophageal Neoplasm

 Patient will be a man

 With a history of weight loss or chronic GERD

 Complaining of progressive dysphagia to solid foods

 Diagnosis is made by endoscopy with biopsy

 Most common type adenocarcinoma

 Comments: usually a complication of GERD/Barrett's esophagus


Diagnosis
1. Barium swallow useful in evaluation of dysphagia. A
presumptive diagnosis can be
made.
2. Upper endoscopy with biopsy and brush cytology is required
for definitive diagnosis.
It confirms the diagnosis in 95% of cases.
Type I and II AC joint separations are managed with a sling and orthopedic follow-up within a
couple of weeks.

They are slightly elevated plaques, gradually turning darker in color, and
appear as if
they were “stuck” on the skin (Figure 11-19).
• Treatment is not necessary and is only for cosmetic reasons: Liquid
nitrogen cryotherapy
or curettage is effective and easily performed in the office setting

A shave biopsy is necessary in order to provide a histologic material for accurate diagnosis and
remove the lesion in a cosmetically acceptable manner. This lesion does not lead to
cancer however a biopsy is needed in order to rule out carcinoma. The lesion is primarily removed
due to cosmetic purposes.

For patients with a first-episode venous thrombosis and documented antiphospholipid antibodies or
2 or more thrombophilic conditions at least 12 months of anticoagulation treatment is indicated.

Deep Vein Thrombosis (DVT)

 Patient with a history of smoking, long-distance travel, surgery, oral contraceptives


use

 Complaining of unilateral leg edema, leg pain, tenderness, and warmth

 PE will show positive Homan's sign


 Diagnosis is made by first ultrasound, Gold Standard: venography

 Most commonly caused by stasis, hypercoagulable state, trauma (Virchow's triad)

 Treatment is IV heparin and switch to warfarin

 Warfarin is contraindicated in pregnancy

 Comments: Risk stratification by Well's criteria

 Question: What precipitating causes may lead to osteonecrosis?

 Answer: Steroids, radiation, or bisphosphonate us

Pulmonary contusion is a common consequence of blunt chest trauma and develops within
24 hours of the injury and resolves within one week. The diagnostic hallmark is finding
irregular, nonlobar opacification of the pulmonary parenchyma on chest radiograph.
Treatment is aimed at pain management followed by chest physical therapy. Complications
may include pneumonia and acute respiratory distress syndrome.

Question: What are the recommendations in regards to a test of cure for a urinary tract
infection in a pregnant patient?

Answer: A test of cure should be performed in all pregnant patients 4 weeks after
completion of antibiotic therapy.
The only populations with a proven benefit from urinalysis and culture screening are
pregnant patients and patients with a planned transurethral resection of the prostate or other
urologic procedure in which mucosal bleeding is expected.
Preform an elliptical excision with 1-3 mm margins
Toxoplasma gondii is an intracellular protozoan parasite that lives throughout the world.
Immunocompetent hosts are infected with the protozoan and typically asymptomatic. Latent
infection can persist for an individual’s entire lifetime. In patients who are immunocompromised, the
infection can reactivate causing disease. In HIV positive patients, this typically occurs only when
the CD4 count has dropped below 100. Patients with CD4 counts at this level have an
approximate 30% chance of reactivating their infection
Toxoplasmosis • Usually a reactivation of latent infection
of Toxoplasma gondii
ut a narrow zone around the vermilion border of the lip is usually spared. Less often, the
perinasal and periorbital areas are sites of significant involvement.

 Treatment is metronidazole and erythromycin


Appendicitis

 Patient will be complaining of fever, pain that began periumbilical then moved
to RLQ, nausea, and anorexia

 PE will show Psoas sign (RLQ pain on extension of right hip), Obturator sign (RLQ pain
on internal rotation of flexed right hip), Rovsing sign (right lower quadrant pain when the left
lower quadrant is palpated)

 Diagnosis is made by ultrasound, CT


 Most commonly caused by a fecalith

 Treatment is surgery

White blood cell count 13,000 cells/mm3 with 76% neutrophils, 10% lymphocytes and 12% bands
he goal of imaging is to make the most accurate diagnosis using the least amount of radiation;
therefore, transvaginal ultrasonography is the imaging modality of choice in the initial
evaluation of pelvic pain.
A 24-year-old female presents to the emergency department with the acute onset of severe
right lower quadrant abdominal pain. Her temperature is 98.5 F. She is doubled over in pain
and has had 3 episodes of vomiting since the onset of pain 2 hours ago. There is no
migration of the patient’s pain. LMP was 2 weeks ago and was normal. She denies any
recent urinary discomfort or abnormal vaginal discharge. Rapid urine pregnancy test is
negative. What emergent study should be ordered on this patient to confirm the diagnosis?

A. CT scan of abdomen and pelvis with contrast


B. Pelvic ultrasound with doppler flow
C. Urinalysis
D. Colposcopy
E. GC/Chlamydia probe

Correct answer: (B) Pelvic ultrasound with doppler flow.

Explanation: Ovarian torsion is a surgical emergency. Pain is classically acute in onset and
severe, often accompanied by vomiting. Torsion occurs when the ovary twists on its pedicle,
cutting off blood flow. Pain is caused by ischemia to the ovary. If this is not treated promptly,
loss of ovarian function can occur. The most frequent finding on ultrasound is ovarian
enlargement. Approximately 50% of women with confirmed adnexal torsion (surgically) will
have normal Doppler flow. Abnormal flow is suggestive of ovarian torsion but does not
confirm the diagnosis. Acute appendicitis should be in the differential for this patient, but the
acute and violent onset of symptoms suggests ovarian torsion. CT scan of the abdomen
might show ovarian cyst, but blood flow to the ovary could not be evaluated so torsion could
be missed.

Acute stress disorder—lasts between 3 days and 1 month. Treatment:


CBT; pharmacotherapy is
usually not indicated.

Cardiac Tamponade

 Patient will be complaining of dyspnea and chest pain


 PE will show muffled heart sounds, JVD, hypotension (Beck’s
triad), pulsus paradoxus

 ECG will show low voltage QRS, electrical alterans

 Echocardiography will show a diastolic collapse of RV

 Treatment is pericardiocentesis
Pulsus paradoxus is an important physical finding in cardiac tamponade, which is the most likely
diagnosis in this rapidly deteriorating patient.

he condition usually follows minor trauma and commonly affects those who take aspirin or other
blood thinners such as coumadin or heparin.

These medications work by antagonizing alpha -receptors in the prostatic urethra and bladder
1

neck leading to smooth muscle relaxation. They are contraindicated in men who are planning
to undergo cataract surgery because they can increase the risk of floppy iris syndrome.

Question: What other class of medications can be used as adjunct with alpha-blockers
in order to treat benign prostatic hyperplasia?

Answer: 5-alpha-reductase inhibitors or anticholinergic agents.


4 mg folic acid once per day

CMV Retinitis

 #1 cause of blindness in AIDS


 CD4 < 50

 Cotton-wool exudates, retinal hemorrhages

 Ganciclovir

 Question: What is the most common treatment of CMV retinitis?

 Answer: Intravenous ganciclovir.

Central retinal vein occlusion (CRVO) (A) generally causes sudden onset, monocular, painless
vision loss.

CMV causes a necrotizing retinitis that if left untreated can lead to blindness. Patients will complain
of decreased visual acuity, visual field defects, floaters and spots, and photophobia. Fundoscopic
examination reveals white, fluffy perivascular retinal lesions, hemorrhages, and necrosis.
Question: What is a risk factor for testicular cancer?

Answer: Cryptorchidism.

Radical inguinal orchiectomy provides a definitive diagnosis of testicular cancer.

Planter fasciitis

Most patients report that the pain usually is most severe during the first few steps after prolonged
inactivity, such as sleeping or sitting but usually resolves upon warming up within 45 minutes.
Patients who are generally standing on their feet all day report that the symptoms may actually
worsen by the end of the day. Physical exam maneuvers that may reproduce the pain of plantar
fasciitis include passive dorsiflexion of the toes, and having the patient stand on the tiptoes and toe-
walk. Radiographs typically are not necessary for diagnosing plantar fasciitis. However, to rule out a
bony tumor or fracture, always consider obtaining at least a plain radiograph before administering a
corticosteroid injection. Treatment is directed at decreasing the inflammation. These treatments
include icing, nonsteroidal anti-inflammatory drugs, rest and activity modification, corticosteroids,
botulinum toxin type A, splinting, shoe modifications, and orthoses.

Question: Which inhaled corticosteroid is most notorious for causing hoarseness?

Answer: Fluticasone.
The lifetime prevalence of hoarseness is 30%. However, only 6% of those who suffer
from it present to a healthcare provider. Hoarseness is defined as any change in the quality of
the voice. Its presence warrants investigation for an underlying cause. Hoarseness results from
ch The lifetime prevalence of hoarseness is 30%. However, only 6% of those who suffer from it
present to a healthcare provider. Hoarseness is defined as any change in the quality of the
voice. Its presence warrants investigation for an underlying cause. Hoarseness results from
changes to the vocal cords. Laryngitis is the most common cause in adults anges to the
vocal cords. Laryngitis is the most common cause in adults

However, patients with cirrhosis have all forms of cytopenia of which thrombocytopenia is the most
common partially resulting from decreased thrombopoietin levels and general bone marrow
suppression.

The patient likely has factitious disorder imposed on self. This is a psychiatric disorder in which
the individual feigns disease in an attempt to gain attention, sympathy, or reassurance. There
is often a longstanding history of doctor and hospital shopping, frequent hospitalizations, and
extensive medical records. They are generally well spoken, intelligent and able to communicate in
medical jargon. They often present on the weekend and after regular office hours in an attempt to
limit access to medical records and personal physicians. Individuals often want to be admitted to the
hospital and once admitted are difficult to discharge home. The normal serum lactate and lack of a
post-ictal state essentially rule out true seizure activity and support the fact the patient is faking his
seizure episodes.
According to the recommendations the hepatitis B
vaccine should be administered to all previously
unvaccinated adults aged 19-59 with diabetes
mellitus, as soon as possible after the diagnosis of
diabetes is made.
Women with an intrauterine device (IUD) who are diagnosed with pelvic inflammatory disease
typically do not require IUD removal regardless of interval since IUD insertion. However, the
exception is in women with pelvic inflammatory disease due to Actinomyces infection. Since
Actinomyces flourishes on foreign bodies, such as an IUD, removal of the IUD in addition to
beginning treatment with penicillin is necessary for treatment and recovery.

Attention deficit hyperactivity disorder (ADHD) is diagnosed in childhood with symptoms of


hyperactivity, impulsivity, or inattention.

In patients with a coexisting tic disorder, the use of an alpha-2 adrenergic agonist (e.g.,
guanfacine or clonidine) may be warranted. Alpha-2 adrenergic agonists are also used when
children respond poorly to a trial of stimulants or atomoxetine, have unacceptable side effects,
or have significant coexisting conditions.

Question: What is the quickest means of obtaining intravenous access in a newborn?

Answer: Cannulation of the umbilical vein.

Evaluating the need for neonatal resuscitation begins with asking: is the infant full-term, does
the infant have good muscle tone, and is the infant breathing or crying? If the answer to any
of those questions is no, further intervention may be needed. Resuscitation begins with
securing the airway, efforts for increasing ventilation and oxygenation, chest
compressions, and finally the administration of epinephrine, volume expansion, or both.
The decision to progress from one step to the next is determined by the response of the infant
to the applied resuscitative effort based upon the respiratory effort and heart rate. Chest
compressions are required if the infant's heart rate remains < 60 beats/minute despite
adequate ventilation for 30 seconds. Chest compressions must always be accompanied by
positive pressure ventilation using 100% oxygen.

A meta-analysis of the efficacy of tests evaluating for the presence of an acute anterior
cruciate ligament (ACL) injury showed that the Lachman test is the most useful.

Question: What are the two sub types of illness anxiety disorder?

Answer: Care-seeking type and care-avoidant type.


Somatization disorder (D) begins before 30 years of age and manifests
as multiple, unexplained physical complaints (four pain symptoms, two
gastrointestinal symptoms, one sexual symptom, and one neurologic
symptom).

Most cases of hemolytic uremic syndrome (HUS) occur in children after a diarrheal
illness caused by E. coliO157:H7. In these cases, the E. coli produces a toxin that damages the
endothelial cells, especially those in the colon and kidney. The first symptoms of the illness are
related to the diarrheal illness and may involve a hemorrhagic colitis as a result of the bacterial
invasion of the intestinal wall. The clinical syndrome involves the development of
a microangiopathic hemolytic anemia and renal failure. The peripheral smear demonstrates a
decreased number of red blood cells (anemia) and fragmented red blood cells as a result of the
microangiopathic hemolytic anemia. These abnormal red blood cells include: schistocytes,
spherocytes, segmented red blood cells, burr cells and helmet cells. Thrombocytopenia is common
as a result of the microthromboses occurring in the syndrome. Fever is often present. The syndrome
mimics thrombotic thrombocytopenic purpura (TTP) in adults.

Question: What is the first-line treatment for an immunocompetent adult with Herpes
zoster?

Answer: Famciclovir or Valacyclovir.


A double dose of the zoster vaccine is recommended for adults aged 50 regardless of whether
they report a previous episode of herpes zoster. A person who reports a negative history of varicella
can still receive the zoster vaccine. The zoster vaccine reduces the risk of herpes zoster (shingles)
by approximately 70%. The vaccine suppresses reactivation of the varicella zoster virus in the dorsal
root ganglia

Sarcoidosis
Biopsies should be performed on the most
accessible lesion that appears to be affected, which
may include cutaneous lesions, subcutaneous nodules, palpable lymph nodes, enlarged parotid
gland, conjunctival lesions, enlarged lacrimal gland, or certain other ocular lesions.

Question: How long is the recommended course of oseltamivir?

Answer: 5 days.
Seasonal influenza is caused by influenza A or B viruses and results in acute respiratory illness.
Clinical presentation includes fever, myalgia, headache and malaise and high-risk patients can
experience serious complications including death. Antiviral therapy can shorten the duration of
illness and viral shedding and decrease the severity of symptoms. Patients with symptoms that
began 24-48 hours prior to presentation should be evaluated for being at high risk of
complications and those at high risk should start a course of oseltamivir. Groups considered to be
at high risk include adults 65 years and older, children 2 years and younger, immunocompromised
patients, pregnant women, residents of nursing homes or chronic care facilities, and individuals with
certain chronic illnesses including chronic pulmonary disease.

Proliferating atypical basal cells with blue-staining nucleus


The patient is suffering from post-partum endometritis for which history of Cesarean section is
the most important risk factor.
In the case presented, the presence of an open internal os defines the miscarriage as inevitable.
Additionally, patients will typically have vaginal bleeding and products of conception can often be
felt or visualized through the internal cervical os.
When anemia is severe (hemoglobin < 10 g/dL), symptoms that warrant additional treatment often
develop. Blood transfusions provide immediate relief of common symptoms, including
dyspnea, fatigue, and dizziness. Risks of transfusions include volume overload, iron overload,
infections, and acute reactions.

The rapid plasma reagin test is a rapid diagnostic test for syphilis antibodies in the serum and
it can be used to rule out neurosyphilis in the patient above. It may be falsely positive in patients
with lupus.

Infectious causes of symptoms resembling mania include herpes


encephalitis, human immunodeficiency virus encephalitis, and neurosyphil
Management of diaper dermatitis involves skin care measures such as more frequent diaper
changing, air exposure, gentle cleansing, and use of topical barrier preparations. The most
common over-the-counter topical barriers contain petrolatum, zinc oxide, or both.

Zinc oxide paste


Acute otitis media (AOM), is also known as suppurative otitis media. Physical examination
reveals a bulging tympanic membrane or middle ear effusion. The diagnosis can be made if
the tympanic membrane has perforated, and the patient has acute purulent otorrhea.
Children with acute otitis media who are < 6 months should be treated with amoxicillin.
Children < 2 years, children with tympanic membrane perforation, and children with recurrent
acute otitis media should be treated for 10 days.
According to the recommendations put forth by the American Academy of Family Physicians, a
peripheral blood smear should be obtained in patients with leukocytosis over 20 x 10 /L who
9

have at least one of the following: 1) associated anemia, thrombocytopenia, or thrombocytosis;


2) enlarged liver, spleen, or lymph nodes; or 3) unexplained constitutional symptoms such
as fever or weight loss. These signs and symptoms indicate that leukemia is highly suspected.

You might also like